Evidence Practice Problems

Pataasin ang iyong marka sa homework at exams ngayon gamit ang Quizwiz!

Problem 2-D: Flight and Guilt As Joe and his assistant Andy are closing the mobile fish and ship stand they operate near a lit baseball field, a man armed with a sawed off shotgun robs them of the evening's proceeds. The next day Joe and Andy examine mug books at the station house and independently identify Carl as the thief. Later that day, the police arrest Carl. At trial, the state calls Brenda. She is Carl's girlfriend, and she answered the door at the time of the arrest. The state offers her testimony that when Carl saw the police approaching, he first ran to the back door, then hid in a closet after discovering an officer standing guard in the alley. Carl objects, arguing that poof of his behavior at the time of arrest is irrelevant. IN a sidebar conference, his lawyer points out that Carl's arrest was based on an outstanding default warrant, issued two years earlier on unrelated drug charges. What is the evidential hypothesis when a prosecutor seeks to offer evidence of flight by the defendant?

The defense would respond to this by saying it was not flight

Problem 14-H: the sick chickens Best Chix, a company that specializes in breeding chickens and selling them to poultry farmers, sues one such farmer, Curt Duval, to recover the balance owed on the purchase price for a flock of chickens. Duval, who claims he is entitled to a price adjustment because many of the chickens were infected with leukosis, seeks to prove this point by testifying to the substance of a veterinarian's report he received, but does not offer the report itself. Best Chix makes a Best Evidence objection. What ruling, and why?

Can farmer testify that he received a veterinarian's report that chickens had leukosis? He cannot speak to the contents of the report, he would need to submit the report

Problem 14-K: the unproduced x-ray In a personal injury suit brought by Sid Landon against Leigh Mills after a two-car collision, Dr. Sherry Nash (Landon's treating physician) testifies for Landon that X-ray of his leg showed a fractured femur. Landon does not offer the X-ray. Mills objects, citing the Best Evidence rule. What ruling? Does it make any difference whether the doctor is relying on the X-ray as part of the basis of a medical opinion rather than merely describing what the X-ray depicts? See FRE 703 and the ACN to FRE 1002.

Doctor offers to testify that x-ray shows broken leg without producing the x-ray ACN to 1002 Rule 703 allows an expert to give an opinion on matters not in evidence and the present rule must be read as being limited accordingly in its application Admissible

"She knew he didn't love her"

Not admit because it is talking about someone else's state of mind

· OJ Simpson Trial

Prosecutor offers evidence that Nicole Simpson told a friend: "I am afraid OJ is going to kill me" --Excluded by trial judge --Not admitted because this suggests that he must have done something to put her in fear --Prosecutor is not offering it for her state of mind, it is about what the defendant has done in the past --Suggests OJ must have done something to her that caused her to have that fear --Backdoor way of getting in hearsay statement that OJ threatened to kill her What if Nicole said "OJ told me that he would kill me if he ever saw me with another man" --Not admit because offered for the truth and the victim cannot be subject to cross examination

Problem 3-A: three see a robbery Higgins is charged with the armed robbery of Bank South. As part of its case-in-chief, the state calls to the stand one Lissner, who entered Bank South shortly after the fact and conversed with three people who apparently saw what happened. The prosecutor proposes to have Lissner describe these conversations, and specifically to testify: That Plaintalk said, with reference to the robbery and to defendant, "Higgins is the one who did it"; That Sirchev said, again with reference to the robbery and to defendant, "That fellow Higgins went out of here carrying money bags"; That Oblique said, once again with reference to the robbery of BankSouth and to defendant, "they ought to put Higgins in jail for this, and throw away the key." The defense objects that the statements by Plaintalk, Sirchev, and Oblique are inadmissible hearsay. Without worrying for the moment whether the statements are admissible, are they hearsay? Why or why not?

"Higgins is the one who did it" --Not admissible because it is hearsay That fellow Higgins went out of here carrying money bags --hearsay, offered to prove that Higgins is the bank robber They ought to put Higgins in jail for this and throw away the key --Do not admit because This is hearsay

"She is planning to file for divorce"

Admit --Hillmon doctrine She was planning to do it, goes to her future conduct

Problem 4-I: drugs across the border Arlen and Bud decide to import cocaine from Colombia. They discuss the matter and agree that Bud will fly there with his friend Carol and acquire the stuff, while Arlen lines up customers. Bud recruits Carol to the conspiracy. Carol's friend Fiona drives them to the airport, and the three share a beer before Carol and Bud board the plane: Fiona: So you guys are going into the import business? Carol: Yeah, only thing I regret is Arlen. He's a real creep. Can he handle his end? Bud: Yeah, he can talk good, and he knows how to set the price. Besides, he's fronted us the buy money for the trip, so what can you say? Don, who works under cover for the Drug Enforcement Administration, approaches Arlen several days later: Don: Looking for some coke. What can you do for me? Arlen: How much you want to buy? Don: Couple or three kilos. Arlen: Hey man, that's a lot of bucks. You good for it? Don: When can you produce and what's the price? Arlen: Got some coming in on Monday. Twenty-five thou per kilo. First-rate stuff. Good price. Don: I need to be able to count on it. Arlen: Hey, you know Bud, don't you? He's gone south to make the buy, and he'll be back on Sunday. You can count on it. Meet you at the Alibi Club Monday at 3. Don: OK. At the airport on Sunday morning, other DEA agents (knowing nothing of Don's contact with Arlen) spot Carol on the basis of their drug courier profile and speak to her as she gets off the plane. Bud is on board too, but he and Carol are traveling separately, and he avoids detection. Finding nothing, the agents release but follow Carol, who finds Bud and tells him, "The feds let me go, and even apologized." On Sunday afternoon, Arlen (followed by Don's colleagues in the DEA) goes to see Bud. On Monday afternoon, Arlen keeps his engagement with Don at the Alibi, but immediately after the sale DEA agents arrest Arlen, and other agents simultaneously capture Bud and Carol at Bud's place. After receiving Miranda warnings, Carol implicates herself and tells the agents that "Bud made the buy in Colombia," and she "just helped carry the stuff past customs down there." Arlen, Bud, and Carol are prosecuted together. All are charged with conspiracy to import and sell cocaine; Bud and Carol are charged with importation of cocaine; Arlen and Bud are charged with possession of cocaine; Arlen is charged with selling cocaine. At trial, the prosecutor invokes the coconspirator exception in offering the following items of evidence: (1) testimony by Fiona describing what Bud told her in the bar (Arlen "fronted us the buy money"), over Arlen's objection; (2) testimony by Don describing what Arlen said (Bud's "gone south to make the buy"), over Bud's objection; and (3) testimony by the DEA agent describing what Carol told him ("Bud made the buy"), over Bud's objection. Do any or all of these statements fit the coconspirator exception?

Prosecutor offers the following statement: Bud's statement to Fiona: "Arlen fronted us the buy money" --Have not established that she is a co-conspirator --The statement was not made in furtherance of the conspiracy, it was just kind of tangential --Do not admit Prosecutor offers: Carole's statement to DEA after arrest: "Bud made the buy" --Not admissible --Does not advance the conspiracy --It is not made during the conspiracy --Once they have been arrested and interrogated the conspiracy is done Prosecutor offers: Arlen's statement to Don: "Bud's gone south to make the buy" --Admissible --Made during the conspiracy --Made in furtherance of the conspiracy

D is prosecuted for contempt of court for failing to appear in response to a summons. D claims he never received the summons. The prosecution calls W, who works at the same office as D, who offers to testify that he saw a copy of the summons on D's desk

Admissible? The existence of the object but also might speak to the content because it would need to know that it was a summons Judicial Notice

Problem 5-H: "It was just a tragic accident" Mandy Hawthorne fell to her death from a scenic lookout in a remote area of Rocky Mountain National Park while on a hiking trip with her husband, Jim Hawthorne. Jim is charged with murder. At trial, the government offers proof that he had taken out a sizeable insurance policy on Mandy's life two months before her death. There is also proof that Jim had a map in the car with an X marking the spot of Mandy's fatal fall. And there is proof that Jim gave conflicting accounts of what happened and behaved peculiarly in other ways in the aftermath of Mandy's death. At trial, Jim testifies that Mandy "fell over the edge when she got too close and apparently lost her balance," and sadly concludes that her death was "a tragic accident" that happened while "we were celebrating our wedding anniversary." In the case-in-rebuttal, the prosecutor offers two proofs. First, there is evidence that a year earlier Jim was working on a deck on their house when he apparently dropped a large joist that struck Mandy and injured her neck. In fact, she narrowly escaped what would likely have been a fatal wound. Second, there is proof that Jim's first wife Linda died more than ten years earlier when she was crushed by a car that dropped off the jack on top of her while she and Jim were repairing a flat tire. The two were alone on a deserted roadside, and Jim had taken out a substantial insurance policy on her life just a month earlier. The defense objects: "This is character assassination, Your Honor. The prosecutor wants the jury to think Jim murdered his first wife Linda, and therefore he probably murdered Mandy too. He can't do that. FRE 404(a)(1) says you can't prove a criminal charge by means of character evidence, and FRE 404(b)(1) says you can't do it with specific acts either. But he is trying to prove that my client has a propensity to kill his wives." The prosecution replies this way: "Your Honor, Mandy's death was no accident, and FRE 404(b)(2) says we can prove prior acts to show that what happened was not accidental. It stretches credulity to say this man suffered the accidental loss of two wives while they were alone with him, and that it just happened that these so-called accidents came shortly after he insured their lives. And the episode with the falling beam that almost killed Mandy stretches the idea of coincidence even further. This proof is proper." Who is right here, and why?

Admit because Doctrine of chances --What are the chances this series of things would continue to happen just by chance? --When that happens that frequently it tends to show a lack of accident Brides in the Bath Case --Murder for killing his third wife and evidence was entered against him that both of his prior wives were also accidentally drowned in the bathtub

Problem 8-D: Faker, Thug Allen is charged with burglary, arising out of an incident in which a masked intruder allegedly gained entrance at night through the ground floor window of a condominium owned by Beatrice and stole jewelry and silverware. State witnesses place Allen in the neighborhood at the time of the offense. Beatrice testifies that she returned home on the occasion in question and surprised the intruder, and in court she identifies Allen as the culprit. But defense cross-examination shakes her testimony, bringing out that in pretrial statements she said the perpetrator wore a "class ring" on his right hand and a "silver banded watch" on his left. (Allen has testified that he wears neither a ring nor a watch, and acquaintances from his place of employment support him on these points.)During cross-examination of Allen, the prosecutor proposes to ask him about an incident four years earlier in which he allegedly falsified his federal income tax return. That event led to a tax fraud prosecution, a conviction on a felony count after a trial, and incarceration for one and one-half years. In the present burglary proceedings, the trial judge holds an in camera hearing in aid of the defense objection. There the prosecutor asks permission to bring out on cross-examination of Allen that. He claimed an exemption on his tax return for a child, though he has no children; He claimed a deduction in the amount of $4,000 for mortgage interest, though he lived in a rented house and was entitled to no such deduction; and He claimed a deduction in the amount of $800 for "charitable contributions," for which he "possessed receipts," when in fact he had no proof of any charitable contributions. Allen objects, arguing that cross-examination under FRE 609 "is restricted to bringing out the fact of conviction, the date and place, and the sentence imposed." The prosecutor replies in this vein: I have no intention of mentioning any conviction. I want to ask about the acts under FRE 608(b). There is no doubt that Allen did precisely what the questions imply, for the record of the proceedings against him establishes these points. There is nothing in Rule 609 that says I must proceed under that provision, and FRE 608(b) authorizes the court to permit cross-examination about "[s]pecific instances of the conduct of a witness" that bear upon "his credibility," as these "instances" certainly do. This man is asking the jury to believe him when he says he did not commit the burglary, and I think the jury is entitled to know that he lied repeatedly on his tax return. How should the judge rule and why? Should the specific limits developed in cases applying FRE 609 apply when the misbehavior of the witness has resulted in conviction, even if the prosecutor proposes to make no mention of the conviction? Can a witness be impeached by prior acts under 608(b) when those acts have resulted in a criminal conviction?

Clash between 608(b) and 609 --cannot use 608(b) as an alternative to 609 There are court decisions that say 609 controls over 608(b)when these two issues are up 609 has limits and if you can always use 608 then you would just go all around 609

Problem 6-C: the jury view In a personal injury action arising out of an automobile accident, a verdict is returned for the plaintiff. After trial, counsel for defendant receives information that two jurors went to the accident scene one evening during the period of deliberations, apparently on a "factfinding" mission. At a hearing on a motion for a new trial, may the defense attorney make inquiry of the two jurors regarding the truth of this report?

May attorney inquire whether two jurors went on a factfinding mission to the accident scene Questioning would be permitted because this would have been extraneous information External influence under 606(b)(2)(B)

Problem 8-B: Hit the deck A man wearing a Halloween mask and carrying a sawed-off shotgun entered Franklin First Bank in Little Rock, ordered customers to "hit the deck," and forced tellers at gunpoint to put cash from their drawers into a canvas sports bag that he was carrying. He then fled the scene. Two weeks later Dan Dennet is arrested and charged with robbery. Ray Elmo is the state's main witness: He is expected to testify that Dennet confessed the robbery to him and sought his help in hiding the money and escaping to Mexico. Dennet is asserting the defense of alibi. He expects to testify that he and his old friend George Farr were on a fishing trip in the Ozarks at the time of the robbery, and he plans to call Farr to corroborate his story. Dennet and Farr have prior bank robbery convictions, arising out of separate incidents. Anticipating that the prosecutor will ask about his prior conviction, which led to a term of three years and ended eight years ago, Dennet makes a pretrial motion to exclude the conviction and bar questions about it. Also anticipating that the prosecutor will question George Farr on his bank robbery conviction, which led to an eight-year sentence from which Farr was released one year ago, Dennet moves to exclude it and bar questions about it.The prosecutor opposes these motions, arguing that "prior felonies are per se relevant" on veracity, and there is no need to inquire into the underlying facts or the trial records. Dennet argues that "felony convictions for robbery have nothing to do with veracity," and the court "should look at the record for insight into what really happened." It turns out that Ray Elmo also has a bank robbery conviction that led to his release from confinement three years earlier. The prosecutor, having argued that robbery convictions are "per se relevant," cannot very well claim the robbery conviction of Ray Elmo is "irrelevant" to impeach him. Suppose, however, the court rules in favor of the defense and blocks questions to Dennet and Farr about their robbery convictions as unfairly prejudicial. Can the prosecutor also object that defense questions to Ray Elmo about his bank robbery conviction should be precluded on account of unfair prejudice?

May Elmo be impeached by evidence of his prior conviction? --Yes, courts are not usually too concerned about government witnesses Prosecution witnesses can usually be impeached by elements of prior convictions --Behavior of witness is not usually under dispute ---ACN to 609 --Probability that prior convictions of an ordinary government witness will be unduly prejudicial is low in most criminal cases May Farr be impeached by evidence of his prior conviction? --This is the alibi witness, they have a prior bank robbery conviction --Court can admit it --This may be prejudicial for the defendant --This is not subject to the special standard --May be subject to the 403 balancing test --The defendant will be prejudiced when the defense witness is impeached ---ACN to 609 --There are cases where a defendant might be prejudiced when a defense witness is impeached May Dennet, the defendant, be impeached by evidence of his prior conviction? --Do not admit ---This is discussion of prejudice ---The jury is not supposed to hear about prior trials ---ACN to 609 In virtually every case in which prior convictions are used to impeach the testifying defendant, the defendant faces a unique risk of prejudice What about the fact that Dennet's conviction was at least 11 years ago --The timeline is based on the date of release

Problem 8-C: A history of lying Ryan Dewald is charged with embezzling union funds in violation of 28 USC §501, a crime that carries a potential fine of $10,000 and imprisonment up to five years. The charges stem from his actions as Treasurer of the Transportation Workers Local 681 in Tampa, Florida, where he allegedly wrote checks adding up to more than $650,000 payable to various law firms for performing legal business for the union. Allegedly these firms submitted inflated bills on Dewald's request and paid him cash kickbacks exceeding $125,000. Wendy Pickett is the principal government witness, and she testifies that she saw couriers from the law firms deliver envelopes apparently containing cash to Dewald. Dewald testifies that the envelopes described by Pickett contained legal documents, not cash kickbacks, and that Pickett is motivated to testify against him in anger over a personal relationship with Dewald that he ended. After Pickett's testimony during the prosecutor's case, defense counsel tells the court and the prosecutor, in a colloquy that the jury does not hear, "this woman has a history of lying," and proposes to ask her about the following on cross: Her misdemeanor conviction nine years ago for displaying a false handicapped symbol in her car in violation of city ordinance, leading to a fine of $500; Her misdemeanor conviction six years ago for petit theft (she shoplifted, leaving a store with expensive women's scarves hidden in the bag with items she had bought, after telling a suspicious clerk she "put the scarves back"). His misdemeanor conviction a year earlier for petit theft (he altered the electrical meter attached to his house, reducing his monthly bills by more than half, and paid a fine of $500). After Dewald testifies during the defense case, the prosecutor tells the court and defense counsel, again in a colloquy that the jury does not hear, "you talk about a history of lying, consider this guy," and he proposes to ask Dewald about the following on cross: His felony conviction eight years earlier for forgery (Dewald signed the name of another official on travel authorizations, and served a year in prison)

May witness be impeached by evidence of her misdemeanor conviction for displaying a false handicapped symbol? --Probably admissible --609(a)(2) says any crime, regardless of consequence ---Involves deceit May witness be impeached by evidence of her misdemeanor conviction for petty theft, where she told the clerk she had put scarves back? --Not admissible ---Even though she told a lie, that probably will not show up in the indictment, the only thing that would show up is that she took the scarves --Not admissible under 609(a)(2) because no deceit --Not admissible under 609(a)(1) because not a felony May witness be impeached by his conviction 8 years earlier for forgery? --Admissible May witness be impeached by his misdemeanor conviction for petty theft for altering an electrical meter attached to his house? --Admit ---The jury could find this was an intention alteration to steal electricity Charging this misdemeanor would have an element of fraud or deceit

Problem 6-D: the bomber Jones is convicted of detonating an explosive device in a public building. Afterwards her lawyer is told that one juror is willing to testify that another informed the jury he was a demolitions expert in the Army and that the type of bomb Jones used was powerful enough to kill anyone within 20 feet, even though no injuries were inflicted by the explosion. If Jones moves for a new trial, may she offer such testimony in support of the motion? If so, may the prosecutor call other jurors to testify that this information had no influence on their votes? May the lawyer put on evidence that a juror shared his special expertise with other jurors?

No, what jurors bring to the juror room is not extraneous because jurors are allowed to bring in their own expertise This would need to be screened out during voir dire

Problem 8-E: He's trying to sandbag us In his civil suit seeking recovery for injuries sustained during an assault, Plimpton claims that Dirk struck him in the chest with a shovel, causing severe injuries. During plaintiff's case-in-chief, Welch testifies for Plimpton that Dirk struck the blow without provocation. Counsel for Dirk then cross-examines Welch, but does not ask whether Welch made any prior statement about the event. During the defense case, Dirk calls Murphy, the police officer who investigated the incident. Murphy testifies that Welch told him that he did not actually see the blow being struck, and thinks Plimpton may have thrown a rock at Dirk. Plaintiff's counsel is immediately on her feet: Your Honor, he's trying to sandbag us. Mr. Welch is gone, and he has had no opportunity to respond to the suggestion that he's changed his story. Defendant had a chance to put the same question to Welch, and deliberately did not. He should have laid a foundation back then. Rule 613(b) requires that Welch have a chance to explain. How should the court rule, and why? Does it matter whether Welch has left the courtroom? What if he was told that he may go home and not told that he remains subject to recall? What if it is learned that he joined the French Foreign Legion and is beyond reach of subpoena? Civil suit for damages for assault

This would not be allowed because the witness is no longer available and cannot respond to or explain the situation 613(b) --The witness has to be around to explain or deny the statement

Problem 6-B: refusal to take the stand Adkins is convicted of unlawful possession of narcotics at a trial in which he did not testify in his own defense. A week after trial, the judge receives a letter from a juror expressing concern that the jury violated the judge's instructions because it considered Adkins' refusal to take the stand as an admission of guilt. The judge informs both counsel of the letter, and the attorney for Adkins files a motion for a new trial based on juror misconduct. May the juror be called to testify on the matters in the letter? May the letter be considered?

To impeach the verdict, defendant offers evidence (testimony and letter) that jury considered defendant's refusal to take the stand as an admission of guilt it is improper to use the failure to take the stand against them jury discussion Inadmissible, that includes the content of the deliberations

Problem 14-J: the unreported burglary In a suit by homeowner Eric Hoskins against Frontier Casualty Company to recover losses suffered when a burglar allegedly stole a valuable Jackson Pollock painting from his home in St. Louis on December 14th, Frontier denies liability and asserts that any coverage that might have existed was lost when Hoskins failed to report the theft to the police. As proof, Frontier offers testimony by its claims examiner Jensen, who would testify that she "looked through the records of reported burglaries in every police station in greater St. Louis for the time period December 14th through December 20th, and found no record of a report by Hoskins and no record of any complaint regarding stolen artwork." Hoskins makes a Best Evidence objection, arguing that "Frontier must produce the logs and records themselves, not just testimony by someone who looked through the written materials." What result, and why? Check the comments in the ACN to FRE 1002 for guidance.

Can insurance company offer testimony by Jensen that she looked through the records of reported burglaries during the relevant period and could find none made by plaintiff? Admissible ACN to 1002 · The rule does not apply to testimony that books or records have been examined and found not to contain any reference to the designated matter

Problem 11-H: Okay, Marie Defendant is prosecuted for extortion in violation of federal law. At trial, the prosecution offers into evidence a note from the defendant to the alleged victim containing the following statements: "Okay, Maurie, this is it, get it and get it straight because you only have one chance . . . . Please Maurie, make it easy on yourself by cooperating fully." The prosecutor requests an instruction under FRE 201 telling the jury that it may, but is not required to, find that in the language of the criminal underworld such statements constitute an implicit death threat. What ruling? Your honor, I request that you instruct the jury that they may, but are not required to find, that the words used by defendant constitute an implicit death threat

Do not allow this · Not beyond a reasonable doubt

Problem 9-C: they saw it the same way I did Shana Lynn sues Dr. Mark Filer and Florida West Hospital for malpractice in Tampa. She claims that he negligently failed to diagnose injuries she had suffered in an earlier diagnostic laparoscopy performed earlier by Dr. Donald Lewis, when Shana went to Tampa Family Hospital complaining of abdominal pain and nausea. In Florida West, Filer ordered a renal ultrasound and scan, which indicated the possibility of urine leaking into the abdomen ("excessive fluid above the bladder"). Filer performed a bilateral retrograde pyelogram, but results were "negative for a urine leak," and he did nothing further. Lynn alleges permanent injuries resulting from Filer's failure to use stents to drain excess fluids. By the time Filer correctly diagnosed Lynn's condition, she suffered internal injuries, infection, and pain. At trial, Lynn's experts (Drs. Debra Miller of Atlanta and Nancy Nielson of Cleveland) testify that Filer breached the standard of professional care observed in the Tampa area. As its expert the defense called urologist Donna Weaver, who described the problems in diagnosing Lynn's condition. Here is the critical part of her direct testimony: Q: How did you determine the appropriate standard of medical care in this case? A: Well I'm a urologist and I practice medicine here, and I'm familiar with Ms. Lynn's condition. I also spoke with eleven colleagues in urology here in Tampa. Q: And at some point were you able to meet with your colleagues? Plaintiff's counsel: Objection, Your Honor. We're happy to hear what Dr. Weaver thinks, but she's about to say what her colleagues think, which is hearsay. Defense counsel: The issue is standard of care, and that's what she's describing. Court: Overruled. You may answer the question. A: Well, I presented Shana Lynn's case in two different forums. One involved five urologists with whom I spoke at our regular meeting. We did what you might call a "curbside consult" and I described the case and asked what my colleagues thought. I also made a formal presentation of her case to six urologists on staff at Tampa State Medical School. These urologists have been in practice here for many years. Q: Based on those consultations, can you say to a reasonable medical certainty whether Dr. Filer met the prevailing standard of care in this area? Plaintiff's counsel: Well, Your Honor, please, she can give her opinion, but not the opinion of others. Court: Overruled, counsel, Dr. Weaver, you may answer. A: Yes, he met the standard. Q: Do you think you have a solid basis for that conclusion? A: Yes, all eleven urologists share my view. They all see it the same way I do. Plaintiff's counsel: Your Honor, please! She can't testify to what her colleagues think. That's blatant hearsay. Court: Overruled, counsel. She's testifying to the standard in Tampa. The jury returns a verdict for the defendant, and the court enters judgment dismissing the claim. Plaintiff appeals. Did the court commit error?

Admissible because it is normal to talk to other doctors to determine the standard of care in the community Defense expert tells jury she consulted with eleven other urologists in forming her opinion and they all see it the same way she does --Not admissible, Getting in the opinion of other doctors without them having to be under oath or testify Defense expert tells jury that she consulted with eleven other urologists in forming her opinion that defendant met the standard of care --Judge would probably allow because there is no suggestion that the other doctors agreed with her "in my opinion, given the testator's mental state, she lacked the legal capacity to make a will" --Not admissible, Have to say what the actual standard for capacity being used is The standard for the expert has to be defined for the jury before it can become a proper question (ACN to 704)

Problem 3-G: Eagle's Rest Bar & Grill Whitney Seaver, Greg Flawn, and Stacey Nichols are charged in United States District Court for the Southern District of Wisconsin with conspiring to distribute cocaine, and related substantive offenses. The case depends partly on showing that the three used a rundown house at 611 Elm Street in Alton, Wisconsin, as a warehouse and occasional living quarters, and that they sold the cocaine at a night spot called Eagle's Rest Bar & Grill in the nearby resort community of Pine Meadows. As is usual in such matters, the evidence was largely circumstantial. Scattered in the array of background evidence offered by the prosecutor were the following bits and pieces: Proof that Whitney Seaver once attended the University of Illinois in Urbana-Champaign, where he (along with many fraternity brothers) was an ardent supporter of his school's football and basketball teams, known in the Big Ten as the "Fighting Illini"; proof that he was known among his friends by the nickname "Witter"; proof that Greg Flawn resided in Alton at 611 Elm Street. The prosecutor had a hard time connecting Seaver with the drug venture, but she offered the following items of evidence on this point: (1) As proof that Seaver had been to the Eagle's Rest Bar & Grill, a book of matches found in his possession bearing the legend "Eagle's Rest Bar & Grill, Pine Meadows"; (2) As proof that Seaver spent time at the house in Alton, a mug found there bearing a large orange capital "I" overprinted with the word "Illinois" and, at the bottom of the mug in block printing, the word "Witter"; (3) As proof that Seaver knew Stacey Nichols, (a) testimony by a barmaid at Eagle's Rest Bar & Grill that she often saw Stacey Nichols with a man whose name she did not know and that, when asked by police officer Isom whether Stacey had ever been there with a man, she (the barmaid) had pointed to the man that Stacey had been with, along with (b) police officer Isom's testimony that the man the barmaid pointed out was Whitney Seaver. To each of these items of evidence, Whitney Seaver has raised a hearsay objection. Are any or all of these items hearsay? Why or why not?

As proof that Seaver had been to Eagle's Rest, prosecutor offers a book of matches found in his possession bearing the legend "Eagle's Rest Bar & Grill" --Two ways to solve the possible hearsay assertion 902(7) by saying just identifying it Testimony by witness—saying something --The court admitted it As proof that Seaver spent time at the house where drugs were sold, the prosecutor offers evidence that police found a mug with the words "Witter" and Illinois --Cannot use 902(7) because it could have come from anywhere --This is purely a circumstantial use that it can connect it to Seaver As proof that Seaver knew Stacey Nichols, testimony of the barmaid that she often saw Nichols with a man, and that she accurately pointed out the man to Isom, and testimony by Isom that the person pointed out was Seaver --The asserted conduct of pointing was done out of court so it is hearsay --ACN states that If the witness admits on the stand that he made the statement and that it was true, he adopts the statement and there is no hearsay problem --If the witness incorporates what they said out of court in court and under oath, there is no more hearsay issue

Problem 12-A: A bum rap After a long publicized trial, Dr. McNary is convicted of slaying his wife and two children, who were stabbed in the family home on October 20th. Dr. McNary claims innocence, and the case against him is circumstantial. His attorney Ashbrooke makes an eloquent argument for clemency, but McNary receives the death sentence. The night after the sentence is announced, Ashbrooke receives a call from Barton, an acquaintance who is also a criminal defense lawyer. On the phone, Barton tells Ashbrooke: I hesitate to call you, but I'm doing it because your client's facing a bum rap. You should know I'm the court-appointed lawyer for a man named Frank Gallo, who is charged with robbing a liquor store on October 20. Just between us, Gallo may be a little crazy. He's been in trouble before, but we don't need to talk about that. He tells me he didn't rob the liquor store, and I believe him because his alibi is too amazing to be false. He told me that at the very moment when the liquor store was being robbed, he—Gallo—was in fact in the McNary house committing what amounts to burglary and murder. Apparently the husband wasn't home, but the wife and kids were, and Gallo told me he killed them with a knife. Now he could have read the story in the paper and made up the part about him doing it, but I've listened to him and frankly I believe him. He even told me where he buried the knife, but I haven't looked for it. I'm not quite sure what to do with this information myself, but I felt I had to tell you about it. The next day Ashbrooke seeks a new trial on ground of newly discovered evidence. At the hearing, he calls Gallo and Barton to the stand. Gallo refuses to testify, claiming the privilege against self-incrimination. When Barton is called, Gallo claims the attorney-client privilege. Should the claim be sustained? What constitutional arguments can be made on McNary's behalf for overriding the privilege? What constitutional arguments can be made on Gallo's behalf?

Can Gallo be required to testify about his possible involvement in the murder? Can Barton testify about the admissions made to him by Gallo? The evidence needed is blocked by attorney client privilege Why doesn't Dr. McNary's constitutional right to present a defense overcome any claim of attorney-client privilege? --Sometimes the constitution will trump the evidence rules Could get Gallo to testify if granted immunity

Rutgers Cyberbullying Trial Rutgers student accused of using webcam to spy on his gay roommate's sexual encounter with another man Gay roommate then committed suicide To prove D used a web camera, prosecutor offers a room request change that victim made to the university housing office stating—"roommate used a webcam to spy on me—therefore I want a single room"

Defendant makes a hearsay objection "I want a single room" --Admit, it is just his statement about his state of mind "roommate used a webcam to spy on me" --Do not admit because it goes beyond the state of mind to say what caused it

Problem 5-J: Ordeal of Leslie or Fred Leslie returns to her sorority sometime after midnight on Homecoming Saturday, earlier than most celebrants. She has been to a luau at the Beta Theta Sigma House, and a few of her more serious friends (who generally avoid such things) discover Leslie sometime later. She is in bed under the covers but not asleep, having taken off her coat and shoes but leaving the rest of her clothes on. She seems tense and sullen but flat in emotional affect and looks at her friends without expression. Ultimately Leslie tells her roommate and a few others that she met a fellow named Fred at the luau. She says she met him several times in the previous summer while she worked at a pool as a lifeguard where he was a swimming instructor. She says she made the mistake of accepting Fred's offer of a ride home from the luau and that he took her to a lonely spot and made advances. At first she "didn't mind," but he "got completely out of hand" and finally raped her. Leslie is persuaded to go to the clinic, where she is clinically examined. Bruises are observed on her legs and forearms, and semen is found in her vagina. As it turns out, Fred is an outsider (a casual acquaintance of Greg, one of the fraternity brothers) who had no business being at the luau. Fred is charged with rape. He pleads not guilty. At his trial Fred claims that Leslie consented, and he defends himself by describing their encounter in ways that, if believed, show that Leslie was a voluntary and even enthusiastic participant in having sex. He also seeks to testify that he and Leslie had consensual sexual relations once during the previous summer. And Fred offers testimony by Greg that "Leslie is sexually very active" and "known as an easy mark." Finally, Fred offers testimony by Thomas (another friend at Beta Theta Sigma) that he had sex with Leslie earlier that same night. The prosecutor objects that the proffered testimony of Fred, Greg, and Thomas is "irrelevant" and "barred by the rape shield" law. Is the objection well taken on either ground? Spell out Fred's argument that the evidence is relevant and the prosecutor's response that it is not admissible. Should such evidence be excluded?

Fred offers testimony by Greg, a fraternity brother, that "Leslie is sexually very active" and "known as an easy mark" --Not admissible Fred offers testimony that they had sex the previous summer --One of the exceptions (b) because it was with respect to the person accused of the sexual misconduct --admissible under (b) Fred offers testimony that Tom had sex with Leslie earlier that night to show source of semen or injury --Not admissible because 412(b)(1)(A) says that there needs to be a dispute as to the actual act and not the consent The defendant admits they have intercourse, does the exception come into play --It doesn't come into play because her sexual history that night is not relevant

Problem 8-F: That's just collateral, your honor Oswald is charged with a robbery in Seattle at 7:00 P.M. on July 14. Defendant claims alibi. As principal witness for the defense, Ardiss testifies that he operates the Jolly Roger Restaurant in Portland, that Oswald is a regular customer, and that he was there for the entire evening on July 14. The prosecutor cross-examines: Q [prosecutor]: To the best of your knowledge, Mr. Ardiss, would you say that Oswald was in the Jolly Roger in Portland every day during the weeks prior to July 14, or was he gone for occasional periods of three or four days, or what? A [Ardiss]: No, it would be my testimony that Mr. Oswald was in there every day during that time. During the state's case-in-rebuttal, the prosecutor calls Detective Kinney of Seattle. Kinney testifies that he saw Oswald in Seattle on June 27 and that "Mr. Oswald told me that he had been there, in Seattle I mean, for the last couple of days before I ran into him." The prosecutor also calls Samuels, a waiter at the Jolly Roger in Portland, and elicits his testimony that "I never laid eyes on Mr. Oswald in the Jolly Roger." Oswald has raised timely objection to the testimony of both Kinney and Samuels: "We have to object to this whole approach. The only thing that counts is Mr. Oswald's whereabouts on July 14th. Where he was on the 27th is beside the point, and nobody cares whether Samuels knew Oswald. That's just collateral, Your Honor, what he's going into now. We strenuously object to these delaying and distracting tactics. It's just not proper impeachment." How should the court rule, and why?

May the prosecutor offers testimony of Samuels, the waiter, that he never saw D in the restaurant on July 14? --Admissible, it directly bears on the date On cross, prosecutor gets Ardiss to say D was at Jolly Roger restaurant in Portland every day for three weeks prior to July 14. --Then prosecutor offers testimony of a Seattle police officer that D was in Seattle on June 27 and admitted to being there for a couple of days ---Not admissible because The lawyer led him down the path of extending testimony from July 14 to more weeks and presence at other times ----This violates the rule about being collateral because it only matters where he was on July 14 and the attempt to impeach alibi witness was incorrect because it led him down the path to impeachment

"She knew he was seeing someone else"

Not admit --Damaging accusation against the husband --Offered under state of mind of wife --Backward looking fact that he is having an affair

Defendant is charged with bank robbery where a .22 caliber rifle was used to threaten the bank teller At trial state offers evidence that police found a .22 caliber rifle at D's apartment although no claim is made that it was the gun used in the robbery

Whether or not to admit depends on the case and the judge Could say no because there is not enough evidence that this was the exact gun used, did not match the serial number or any other distinguishing factor

Problem 6-F: Woman drivers A female defendant was prosecuted for vehicular manslaughter in connection with an accident on a snowy road that killed two occupants of a vehicle with which she collided. At trial, the principal issue was whether the defendant was driving in a reckless fashion or whether the wintery conditions were responsible for an unavoidable and tragic accident. After deliberating for several days, the jury of ten men and two women sent the judge a question, inquiring what to do if the jury could not agree on a verdict. After consulting with counsel, the judge gave the jurors a traditional Allen charge, encouraging them to continue deliberating in good faith and to consider the views of those jurors opposed. The next day, the jury returned a guilty verdict. The trial judge polled the jury and released them from service after admonishing counsel not to contact the jurors. Three months later, defense counsel moved for a new trial and provided affidavits the defense obtained from the two female jurors in support of the motion. The two jurors stated that they had believed the defendant's testimony and felt firmly that she was not guilty of the offense and held out against conviction for several days. After the court's Allen charge and on the final day of deliberations, another male juror had an outburst, in which he berated the two holdout jurors for refusing to vote to convict, exclaiming, "We all know that women drivers are the worst! This gal's incompetence behind the wheel got two people killed and you don't want to admit it because you are trying to protect your sister." Although the women denied the charge, they voted to convict that same day. The jurisdiction has a no-impeachment rule that mirrors FRE 606(b) and the prosecutor argues that this evidence regarding internal deliberations is prohibited. The defense argues that the defendant's constitutional right to an impartial jury was violated by the sexist remarks and that Peña-Rodriguez requires the trial court to hear testimony from the jurors in support of a motion for new trial. How should the court rule? What effect, if any, should it have on the outcome that defense counsel ignored the trial judge's admonition not to contact jurors? Defendant seeks to impeach verdict on two grounds

Would try to extend Pena Rodriguez beyond racist remarks

Witness states under oath in deposition that the defendant killed the victim. At trial witness testifies the defendant did not kill the victim. Witness prior inconsistent statement is admissible to prove the truth of what it asserts

Admissible because it is a prior inconsistent statement and did not need to be under oath

D is prosecuted for the murder of V, his girlfriend. V was shot to death in D's apartment. D's defense is that V was at his apartment and requested to see his gun collection, that he handed her a pistol and that while she was examining it she dropped it and it went off, killing her. The state calls V's girlfriend to testify that a week before the shooting V told her that she was deathly afraid of guns and was upset that D collected them

Admit --It is relevant because it rebuts his alibi If she told her friend she is terrified of gun, it shows a state of mind that she probably would not have asked to see the gun

Problem 5-D: What price Truth? In Don's trial, the judge excludes the testimony of Coach Jones during the prosecutor's case in chief and admits the testimony of Reverend Gram during the defense's case in chief. After Gram has told the trier of fact that Don is a peaceable nonviolent fellow, the prosecutor rises for cross examination. During cross, Reverend Gram was asked if he knew that Don's wife was treated in the emergency room suffering injuries from Don

May the prosecutor ask on cross whether Reverend Gram knows that Don's wife was admitted to the emergency room two weeks ago? --If you put a character witness on, then the other side can bring out evidence and information about relevant acts that could not have been brought out before as long as it is within the scope of the direct

P is injured in 2017 from an accident resulting from a steering defect in his 2014 car The manufacturer made a design change to correct the defect in 2016

P wants to offer evidence of the 2016 design change --Admissible The triggering time is when measures taken would have made an earlier injury not occur 407: subsequent remedial measures --When measures are taken that would have made an earlier injury or harm less likely to occur, evidence of the subsequent measures it not admissible to prove § Negligence § Culpable conduct § A defect in a product or its design; or A need for a warning or instruction

Problem 14-M: the tax evader Brad Trimble is prosecuted for alleged tax evasion, and the government calls Charles Urban, an IRS accountant, to testify. Urban has examined Trimble's bank records, which reflect more than 90 deposits and 300 withdrawals over the year, and the prosecutor offers (1) Urban's testimonial summary of deposits and disbursements from the account and (2) a chart prepared by Urban from bank records summarizing entries that the prosecutor considered significant. Trimble raises a Best Evidence objection. The prosecutor invokes FRE 1006. What result?

Should the testimonial summary be admitted? Testimonial summaries are admissible Evidence from FBI agents and the like have to be in the form of testimonial summaries Should the written chart summary be admitted? · Admissible 1006 Proponent may use a summary, chart, or calculation to prove the content that cannot be conveniently examined in court Documents underlying a summary must be admissible but they do not have to be admitted into evidence Can testify to the existence of something without production of the original, but cannot testify to the content

Problem 13-K: the house of the rising sun Under the headline "Mayor Linked to Prostitution," the Daily Post ran a story stating that Mayor Cook was found to have a one-half ownership in the House of the Rising Sun, a local prostitution establishment. The story appeared under the by-line of Ron Bellamy, who was identified as "staff reporter." A week later, a letter to the editor was published in the Post, signed by George Ramsey, congratulating the Post on its exposé of the mayor and stating that "Cook has been the prostitution kingpin in this town for years, and it is time the public knew about it." Mayor Cook files a libel action against the Post, Bellamy, and Ramsey. Bellamy defends on the ground that the published version of the story was not the same as the version he submitted to the city editor. Ramsey defends on the ground that he never wrote the letter in question. The Post settles prior to trial. At trial Mayor Cook offers as exhibits the two issues of the Post where the story and letter appeared. Bellamy and Ramsey object on grounds of lack of authentication. What ruling? Is the newspaper sufficient evidence to establish that Ramsey was the author of the letter to the editor?

No ACN Establishing authenticity of publication may still leave open questions of authority and responsibility for items therein contained

Problem 14-F: the surveillance photograph During a bank robbery, a hidden surveillance camera takes digitized pictures of the robber. At trial the bank security officer, who was not present at the robbery, testifies that he downloaded the digitized files that were created automatically at the time of the robbery, and he examined the image of the robber. He offers to testify that the person shown in the image is the defendant. The image is not offered in evidence. Defendant makes a Best Evidence objection. What ruling?

Security guard offers to testify that person shown in image is the defendant Not admissible This is optional because they could prove that it was the defendant through other means But once the party choose to use pictures, it has to be the original

Problem 5-L: I told him to stop In his federal trial for the alleged rape of Karin on Fort Linden Air Force Base at 1:30 a.m. on Sunday, Craig claims consent. He is 32 years old, and works at Fort Linden as a civilian avionics technician and instructor. He resides in an apartment in town. Karin is a 27-year-old Air Force enlisted woman in the avionics training program. She is enrolled in a course that Craig teaches, and lives in barracks on Fort Linden. Government proof shows that Craig and Karin met at the Aero Squad Bar, located in town. While there, Karin and Craig drank gin and tonics for several hours. Karin had come to the bar with other women from the base, but they left on the understanding that Craig would drive her back to Fort Linden. Karin testified that on arriving at the base Craig suggested that they "go up the hill" to a secluded area of trees and grass that served as a picnic ground for base personnel. The two sat on a blanket that Craig produced from his trunk and continued to drink gin from a pint bottle he had bought at the package store before leaving the Aero Squad. Karin testified that Craig "put his arm around me and we kissed," but that she "physically resisted" when "he began touching me through my blouse and started to unbutton my blouse and skirt." She continued, "I told him to stop and that I didn't want to have sex with him tonight," but he "kept pressing himself against me and trying to reach up my blouse which had come untucked," until "I finally stood up and told him I was going to walk back to the barracks, and I proceeded to run as fast as I could." At that point, according to Karin, Craig dragged her back to the blanket, where he held her arms down with his knees, ripped off her clothes, and raped her. On cross, Craig's lawyer tries to get Karin to admit she was "laughing and responding in kind" when Craig kissed her, and wasn't "resisting in any way" and in fact "co-operated in taking off" her clothes. Karin agrees that they were "laughing at first" and that she kissed him, and even helped unfasten her blouse and bra because she was "afraid he was going to rip the fabric," but she emphatically denies "helping him take my skirt off," and denies "cooperating when he raped me," saying she "told him to stop" and "tried to get him off me, but he was too heavy, and held my arms down." Anticipating Craig's testimony, the government proposes two further proofs that what happened was rape rather than consensual sex: First, the prosecutor calls Laura. In aid of an offer of proof, she testifies outside the jury's hearing that she "dated Craig for several weeks" a year earlier while they were living in another state, and that once when they returned to her apartment after a movie, Craig "tried to rape me." She "made the mistake" of letting him touch her, and he "kept coming on after I told him to stop," and "tried to rape me," except that she "screamed and kicked him in the privates" so hard that he "cried out in pain" and ran away. Second, the prosecutor offers a certified copy of a judgment of conviction for sexual assault on a minor obtained against Craig in another state three years earlier. The victim was 13-year-old N, daughter of a woman named Rita, with whom Craig was living at the time. The record indicates that Craig encouraged N to drink beer one afternoon while Rita was at work, that they then played strip poker, and that Craig fondled and sodomized N. He served a year in prison and was on probation at the time of the charged offense at Fort Linden. Counsel for Craig objects to these proofs: Laura's testimony and the prior conviction amount to character assassination. Her story cannot be admitted to prove intent or modus operandi because there is no dispute about the identity of the alleged assailant and the situation is too dissimilar from what supposedly happened at Fort Linden between Craig and Karin. And the conviction for abusing N shows acts of a totally different nature that cannot be relevant in any way to what the government claims here. If this evidence is admitted, the jury will be invited to convict the defendant just because it thinks Craig is "that sort of person." These proofs will inflame the jury, inviting it to convict Craig out of anger at what he may have done before, which is classic unfair prejudice. How should the court rule on this objection under FRE 413 and FRE 403, and why?

Prosecutor calls Laura who offers to testify that once while on a date with Craig he tried to rape her even though She has never previously reported this incident --Admit, it is not relevant that it was never reported and this is the first time she talked about it Is determining whether prior assault occurred a 104(a) or 104(b) question --104(b) Prosecutor wants to offer certified copy of a judgement of conviction for sexual assault on a minor earlier --There are arguments against --Would not be as relevant because they were different ages (401) --It was done with a minor so it is more prejudicial (403) Can Laura's testimony be excluded under 403? --There would be a greater argument to use 403 --Need to show how probative the evidence is and how prejudicial it is

Problem 11-C: Interstate Call Lauro is charged with knowingly using the telephone in interstate commerce for purposes of transacting bets or wagers. Telephone records show the call at issue was placed interstate from Baborian in New Haven, Connecticut, to Lauro in Rhode Island, but Lauro denies knowing that it was an interstate call. At trial the only evidence of knowledge was a transcript of an intercepted telephone call at 6:36 p.m. from Lauro, who admittedly was in Rhode Island, to Barborian's father, who was in New Haven, Connecticut. In that call the father said to Lauro that Barborian was with him in New Haven at that time, but was driving to Rhode Island and would call Lauro shortly. At 6:51 p.m., Lauro got the call from Baborian that is the subject of the prosecution. The prosecutor asks the court to take judicial notice (1) that the driving time from New Haven, Connecticut, to Rhode Island is more than 15 minutes and (2) that therefore Lauro must have known Baborian's call was from out of state. What ruling?

Your honor, I request that you take judicial notice of the following fact: that the driving time from New Haven to Rhode Island is more than 15 minutes Admissible, a matter of geography Your honor, I request that you take judicial notice of the following fact: because the driving time from New Haven to Rhode Island is more than 15 minutes, and defendant received the call only 15 minute after the caller left New Haven, defendant must have known it was an interstate call Not admissible ·Courts are reluctant to notice what is on somebody's mind

In a burglary prosecution, a witness testifies that he noticed a car parked near the scene (turned out to be D's car) because it had three parking tickets on the windshield. D objects that this testimony is improper because the prosecution did not produce the tickets

Admit, this is testifying to the existence of the tickets and not to their content

Problem 4-P: He had nothing to do with it While driving an 18-wheeler north on I-55 near Bloomington, Illinois, John Garvin is stopped by State Trooper Howard for a Motor Carrier Code violation. Garvin gets out of the cab and produces his license but does not have a logbook or bill of lading. On request, he lets Howard search the cab for these items, telling them he is traveling with co-driver Will Torrens, who is resting in the bunk behind the seat. Officer Howard asks Torrens to get out of the cab and notes a strong smell of air freshener as Torrens emerges. Howard then looks through the cab. Once inside, Howard smells a strong odor of marijuana and sees four brown suitcases, one with a sidepocket that is slightly unzipped. Howard sees duct tape through the opening and feels "bricklike objects" when he squeezes the bag. Howard radios for backup, and a trained dog is brought to the scene, leading to a search and the discovery of marijuana, which is packed in the suitcases and hidden behind the bunk in the sleeping area. Howard reads Garvin his Miranda rights. Garvin tells Howard that the company assigned him to work with Torrens when Garvin's truck broke down, that the two drove from Milwaukee to Dallas to deliver a load of cheese, that on arrival in Dallas they got a hotel room, that Torrens left for several hours and on his return said the company wanted them to drive the cheese to Kansas City. Thereafter Garvin and Torrens took turns driving, and Torrens told Garvin the cheese had been delivered in Kansas City while Garvin was sleeping. Garvin said he never saw the bill of lading and knew nothing about any marijuana. Thereafter Howard reads Torrens his Miranda rights and they talk outside the hearing of Garvin: Torrens tells Howard: "The marijuana belongs to me, not Garvin. He had nothing to do with it. I just wanted to get rich quick. I took a chance and lost, and now I have to do the time. I'm ready for it." Garvin and Torrens are charged in federal court with possessing marijuana with intent to distribute. Their motion to suppress the marijuana is rejected, and Torrens pleads guilty, but Garvin does not. In Garvin's trial, Torrens refuses to testify. The government proves the truck was empty and argues that Garvin must have known that no cheese was ever carried or delivered and must have smelled the marijuana because its odor was apparent to Howard despite the air freshener. Garvin calls Howard to testify to what Torrens said about the marijuana belonging to him and Garvin having "nothing to do with it," invoking the against-interest exception. The court refuses to allow this testimony, concluding that the exception does not reach the part of the Torrens statement exonerating Garvin. Ultimately Garvin is convicted. On appeal, he argues that the court erred in excluding Torrens' statement to Howard. What result, and why? Torrens tells police officer—"The marijuana belongs to me. Garvin had nothing to do with it"

Admissible as a statement against penal interest? --Admit because they were not friends --Look to corroborating circumstances If they were friends then it might have been false

Problem 14-E: the XXX rated movies In a trial for alleged interstate transportation of obscene films, the prosecutor tries to establish the obscene content of the films seized from the defendant by the testimony of a police officer who viewed them. The films themselves are not offered into evidence or shown to the jury. The attorney for the defendant makes a Best Evidence objection to the police officer's testimony. What ruling?

Police officer who viewed films offers to testify to their obscene content Not admissible Movies count as movies and they should have to be shown to prove the content

Problem 4-B: he thinks I'm his wife Martin Trask is tried in federal court in New Mexico for the alleged abuse of his four-year-old daughter "S.T." occurring on the Navajo Indian Reservation there. S.T. had been staying with him under an agreement giving him primary custody when he and S.T.'s mother Bonnie divorced. The summer after the divorce, S.T. went to live with her mother Bonnie in Colorado for a month. During this visit, she told Bonnie that Trask had sexually abused her. Bonnie took S.T. to be examined by a pediatrician and interviewed by a social worker. S.T. made detailed statements to both, describing the abuse and telling the social worker that Trask "gets drunk and thinks I'm his wife." Authorities were notified in New Mexico, and criminal charges were filed against Trask. At trial, the prosecutor called S.T. to testify (she was then six and a half), but she proved reluctant to describe the alleged abuse. All the prosecutor could get from her was "yes" or "uh huh" to a leading questions, such as "Did your dad touch you in your private place?" The trial court commented on the "very difficult situation" before it. Martin Trask's lawyer tried to conduct a gentle cross-examination, anxious to avoid the appearance of meanness to an apparent abuse victim, but he eventually asked this question: "Isn't it true that you are just making up this story about your dad because you would prefer to live with your mother in Colorado?" In rebuttal, the government sought to refute the suggestion that S.T. had just made it all up. Invoking FRE 801(d)(1)(B), the prosecutor called Bonnie, the pediatrician, and the social worker to testify to the more detailed statements S.T. had made, describing the abuse. If allowed, these three witnesses would recount S.T.'s comment about Trask getting drunk and thinking S.T. was his wife, and the pediatrician and social worker would recount more specific statements (that Trask touched S.T.'s vaginal area).In aid of an offer of proof, the prosecutor argues thus: Your Honor, what S.T. told her mother, the social worker, and the pediatrician are admissible as consistent statements under FRE 801(d)(1)(B), which means they can be used as substantive evidence and to repair credibility. They are consistent with her testimony here, in which she answered in the affirmative when I asked her whether Martin Trask had touched her vaginal area. They are admissible because the defense has seen fit to try to impeach S.T. by suggesting that what she's saying is just part of an attempt to be with her mother in Colorado permanently, and that it's all made up. In support of its hearsay objection, the defense argues thus: Your Honor, S.T.'s statements to her mother, the pediatrician, and the social worker are classic hearsay. They don't fit Rule 801(d)(1)(B)(i) for two reasons: First, what S.T. said before doesn't refute our contention that she's making up this story so she can stay with her mother in Colorado. Only statements made before the motive arose can refute claims that her testimony is motivated by that interest, and these statements were made after she decided she wanted to stay with her mother in Colorado. Second, the Rule requires consistency. Oh sure, you can say that multiple detailed accounts of alleged abuse are consistent with a one-word answer to the question "Were you abused?" But the devil is in the details, and the prior detailed statements are everything in this case. They go far beyond anything S.T. testified to. The exception was never designed to get in the only convincing evidence in the case, but for the limited purpose of repairing credibility on the points to which the witness has actually testified. How should the court rule on these objections, and why? "Aren't you making this story up because you want to have custody changed so you can continue living with your mother?

Prosecutor offers allegations that child made to mother, pediatrician, and social worker Not admissible --The statements were made after the alleged motive ---During the summer developed preference to stay with mother Tome case requires the pre-motive statement

Problem 1-A: how did it happen? At the intersection of Folsom and Valmont, two cars collide—a yellow Fiat driven by Abby Barton, in which Carl Dreeves rode as passenger, and a blue Buick driven by Eric Felsen. In a Rules jurisdiction, Barton sues Felsen for personal injuries and property damage. During her case-in-chief, Barton calls Carl Dreeves, who testifies on direct examination that "the Buick ran a red light." On cross-examination, Felsen's counsel asks the following questions: Q: Now Mr. Dreeves, you and Ms. Barton are seeing each other socially, isn't that right? Q: Isn't it true, Mr. Dreeves, that at the time of the accident Ms. Barton here had turned clear around in her seat and was looking out the back window of the car? Q: Tell me, Mr. Dreeves, you and Ms. Barton here had just finished lunch at Sebastian's where she drank three glasses of wine just before the accident, isn't that true? To each question Barton's counsel objects, "Improper as beyond the scope of direct, Your Honor." How should the judge rule in each instance, and why? What arguments do you expect from Barton and Felsen?

You are seeing each other socially, isn't that right? Not admissible if it does not affect the witness credibility

Problem 4-O: The government let her go Rick Masters is tried for importing cocaine when he and a 17-year-old Australian woman named Jane Shell are arrested as they arrive in Puerto Rico on a plane flight from Peru. Her name resulted in a "hit" during a computer check of debarking passengers, meaning that the Drug Enforcement Agency had information that she and an associate were suspected of smuggling drugs. A body search of Shell resulted in the discovery of ten packages of cocaine attached by tape inside her undergarments. Agents arrested Masters for the same reason, but a search of his person yielded no contraband. Shell was arrested and detained in an adult prison facility, and she gave a statement that led to the indictment of Masters. Several weeks later, the U.S. Attorney sought permission to take Shell's deposition, suggesting that she might return to Australia and it would be impossible to bring her back to testify against Masters. The court permitted the deposition over defense objection, and defense counsel attended but did not question Shell. She incriminated Masters, testifying that he "hired me to carry the cocaine, which I was to turn over to him in exchange for $500 and my airline ticket to Australia when we arrived here in Puerto Rico." Shell also said she would not appear at the trial of Masters, and the U.S. Attorney returned her plane ticket and passport. In the trial of Masters several months later, the government offers the deposition testimony of Shell, invoking the former testimony exception. The U.S. Attorney explains that "we checked with our Embassy in Australia and they found Shell, but she refuses to come back." The defense objects: If Shell is unavailable, Your Honor, it's only because the government let her go. Indeed, they practically invited her to go by returning her ticket and passport. In the words of FRE 804(a), Shell is not unavailable because her absence is due to the procurement or wrongdoing of the government, and their purpose here was just what the Rule says—preventing the witness "from attending or testifying." The U.S. Attorney rises to her feet and argues the other side: Your Honor, that provision contemplates at least negligent misbehavior and perhaps something worse, like deliberate scheming. We didn't do anything wrong. We couldn't just hold Shell in jail or deny her right to return home for two months. We knew she would likely leave, so we took her deposition with your permission. The defense was there and could cross-examine. Should the court admit Shell's deposition? Why or why not?

If the government sent her back to Australia or did not attempt to make her available in any way then the statement could not be admitted (804(a)(5)) Court said the government could have done something more to make sure she could be in court

Problem 9-B: the watchful neighbor A pickup truck driven by defendant Al Davis collides with a car driven by Sandy Pinkston, seriously injuring her child Amy, who is in the right front seat. In the ensuing lawsuit by Sandy Pinkston against Al Davis, plaintiff calls Luke Hanson as a witness. Peruse the following transcript of Hanson's testimony on direct and cross-examination. Consider whether either side has proper objections under FRE 701. Consider too how you would avoid or cure any problems in Hanson's testimony. Direct Examination Q [plaintiff's counsel]: Please state your name and address for the court reporter. A: Luke Hanson, 1623 Elm Street, in Fayetteville. Q: Mr. Hanson, directing your attention now to the afternoon of June 12 of last year, did you see an automobile accident in the 1600 block of Elm Street, near your home? A [Mr. Hanson]: Yes, sir. I was sitting outside on my front porch swing. It happened practically in front of me. Q: Please tell the jury what you saw. A: Well, the lady across the street, Mrs. Pinkston, was backing her car out of the driveway to take her daughter to ballet lessons when a pickup truck plowed right into the side of her. Q: Did you see the driver of the pickup? A: I did. Q: Do you see him here today? A: I'm really not sure. It's been almost a year. The driver had a beard and that fellow is clean-shaven. But I would guess that's him over there [pointing to the defendant]. Q: Let the record show that the witness pointed to the defendant. Now Mr. Hanson, how fast was the pickup going at the time of the collision? A: I would say he was going at least 35 miles per hour. Q: And what is the speed limit on that street? A: Well, generally it's 35 miles per hour, but the law says you can go only 20 if you're within 500 feet of a school, and we are. So he was breaking the speed limit, no question about it. Q: Did you go over to the pickup after the accident? A: Yes, sir. Q: Did you notice any unusual odor coming from the truck? A: Yes, sir. There was a strong smell of pot. I'm sure the driver was smoking a joint—had been, anyway. Q: Would you say he was stoned? A: I really can't say. Q: Well, how did he seem? A: It's hard to say, but he had a real guilty look, like he was afraid he was going to lose his license or get sued. Q: What was Mrs. Pinkston doing after the accident? A: She was crying, real hard. Q: What about? A: I think she was upset about her little girl Amy, who got hurt the worst of anyone. Q: What injuries did Amy and her mother suffer? A: Well, Amy looked like she had a broken back, but Mrs. Pinkston only had a dislocated shoulder. Q: How extensive was the damage to the station wagon? A: The whole right side was caved in. The car was totaled. Q: And do you know the approximate value of the car? A: I'd say about $5,000.Q: No further questions. Thank you, Mr. Hanson. Cross-Examination Q: Mr. Hanson, isn't it true that Mrs. Pinkston barreled out of her driveway without stopping or looking to see whether anyone was coming? A: Well, she looked like she was in a hurry, and she didn't turn around to look at the street as she came out. Q: And Mr. Davis, in the pickup, did everything he could to avoid a collision, isn't that so? A: Yes, he couldn't have stopped in that short a space. Mrs. Pinkston just backed out right in front of him. It was sad to see. Q: Now Mr. Hanson, let's assume that we can identify the most careful driver in the world. If he or she had been driving along Elm Street on that fateful afternoon, could that hypothetical perfect driver have avoided Mrs. Pinkston's car, backing out the way she did? A: No, I don't believe so.

"I would guess that is him over there" --Most courts will allow some qualification and witnesses rarely say they are completely certain "I would say he was going at least 35 miles per hour --Most courts would admit an estimate "the law says you can only go 20 miles per hour if you are within 500 feet of a school, and we are, so he was breaking the speed limit" --This is not admissible because the court instructs the jury on the law "there was a strong smell of pot" --Admit because this is common enough that a law witness could testify to its smell "he had a real guilty look, like he was afraid he was going to lose his license or get sued --Not admissible because it is speculative "what was Mrs. Pinkston crying about" "I think she was upset about her daughter" --Do not admit because it violates 701 because it is getting into someone's head "Amy looked like she had a broken back, but Mrs. Pinkston only had a dislocated shoulder" --Not admit because Looking at someone, you cannot tell what their injuries would be or were "the car was totaled" --Not admit because Would require the neighbor to know the current market value and cost to repair the car "the approximate value of the car was 5,000" --Not admit because only the owner of the property can give opinion on the value o "D did everything he could to avoid a collision --Not admit, this is speculative and the neighbor would not have been able to know that "could a hypothetical perfect driver have avoided the collision? --Not admit, Goes too far and gets too speculative

Declarant says to her girlfriend: "you ought to see all the dope that D and I have in our hotel room"

Do not admit because this is just chatter and does not further the conspiracy

Problem 8-G: She handed me the heroin Clair and Arla are suspected of conspiring to distribute heroin. An informant introduces undercover FBI Agent Turner to the two women, and during lunch at a restaurant Clair agrees to sell Turner 100 grams of heroin. Turner suggests that Clair go to the women's restroom to put a sample of the heroin in a paper towel so Turner can test it. Both women go to the restroom and later return with the heroin, and the sale is made. Clair and Arla are arrested right outside the restaurant and charged with selling heroin. Prison personnel later perform a search and discover that Clair (not Arla) is carrying a large quantity of heroin. Thereafter Clair pleads guilty to a drug offense. Eighteen months later Arla goes to trial on the charge of selling heroin. Turner testifies for the government that when the two women returned from the restroom, it was Arla who produced the heroin from her purse, wrapped in a paper towel. Counsel for Arla vigorously cross-examines: Q: Isn't it a fact, Mr. Turner, that you really don't remember which woman was carrying the heroin because you were focused more on the women than on the drugs? A: No, sir, I remember very well. It was the woman in the black and white dress Q: Aren't you really just trying to put two women in jail, and you think my client is guilty because she was with Clair—in effect, mere guilt by association? A: No, sir.During the defense case-in-chief, both Clair and defendant Arla testify that it was Clair who carried the heroin back and handed it to Turner. During the government's case-in-rebuttal, the prosecutor offers a recorded statement by Turner, made outside the restaurant moments after Clair and Arla were arrested, in which Turner said: "The woman in the black and white dress had the stuff in her purse. She took the heroin out of her purse after she came out of the restroom and handed it to me." It is undisputed that Arla was wearing the black-and-white dress, while Clair wore slacks and a blouse. Arla raises a hearsay objection, and argues in addition that the prosecutor is engaging in "improper rehabilitation." How should the court rule, and why? After Clair pleads guilty to distribution of heroin, Arla goes to trial. Turner testifies it was Arla who sold him the heroin

Just need to determine when the tape was made to determine if the was promotive or postmotive If the tape was made after Clair pleaded guilty --Post-motive because there was a motive to incriminate the other person After the drugs were found on Clair --If it was before Clair was charged, it would be premotive the court determined it was premotive because there was no point in trying to incriminate Arla when the tape was made

Problem 14-L: testimony v. photocopy In a breach of contract action, Corrigan, the plaintiff, establishes that the original written contract was destroyed in a fire through no fault of his own. Even though Corrigan has an accurate and legible photocopy of the original contract, he chooses at trial to prove the terms of the contract by his testimony rather than by offering the photocopy. Defendant Gregor makes a Best Evidence objection to the testimony, arguing that the photocopy must be introduced. What ruling? See FRE 1004 and the ACN to FRE 1004.

After proving that original written contract was destroyed in a fire, plaintiff offers to prove the contract by testifying to its terms rather than by offering a photocopy Admissible ACN to 1004 Rule recognizes no degrees of other evidence The original is gone but there are no hierarchy between using the testimony or photocopy as the next best thing

Problem 4-M: Where did she fall? On leaving Hagen Grill in Chicago after dining with friends one evening, Doris Welsh felt woozy. The next thing she knew, she awakened in a bed in nearby Emmanuel Hospital. Treating physician Paul Thorne found that Welsh was suffering from a broken hip and dislocated shoulder, apparently from a bad fall. Welsh sued Hagen Grill, alleging that a defect in the stairs leading out of the restaurant had caused her to fall. Measurements indicated that the height of risers in the stairs was 8¾ inches, almost two inches more than the standard 7 inches. Welsh didn't remember how she got to the emergency room, but a passerby named Greg Shaw called 911 on his cellphone. After an ambulance arrived, Shaw accompanied Welsh in the ambulance to the hospital. Consider the following proffers: For plaintiff, the ambulance driver would testify that "Ms. Welsh was unconscious when he first arrived, but as she came to she complained of excruciating pain in her hip and right shoulder." Asked by plaintiff's counsel whether Welsh said anything about how it happened, Dr. Thorne would testify "She told me that she'd fallen on the stairs in the restaurant." For defendant, the hospital intake specialist would testify that "Mr. Shaw brought Ms. Welsh to the ER, and told me he saw her trip on the sidewalk in the park near the plaza east of Hagen Grill." On proper objection, should the ambulance driver, Thorne and the intake nurse be allowed to testify as indicated? Do their statements fit FRE 803(4)?

Ambulance driver offers to testify that "Ms. Welsh complained of excruciating pain in her hip and right shoulder" --Admit because it is medical diagnosis and can be made to anyone medical and does not have to be made by the injured person The hospital intake specialist offers to testify that Mr. Shaw (the passerby who took Welsh to the hospital) told him "that he saw her trip on the sidewalk in the park near the plaza east of Hagen Grill" --Not admit because The doctor doesn't need to know where she fell --Location cannot come in in this instance Dr. Thorne offers to testify that Welsh "told me that she'd fallen on the stairs in the restaurant" --Not admit --Where someone fell is not something the treating doctor needs to know --She is not a party opponent --She cannot offer her own statement

Problem 11-G: deadly weapon Stimson, a karate expert, is charged with "assault with a deadly weapon, to wit, his hands," arising out of an attack on Boyer. Under the law of the jurisdiction, whether a weapon is deadly under the statute is an adjudicative fact for the jury. But on request by the prosecutor, the court instructs the jury under the jurisdiction's identical counterpart to FRE 201(f) that "you may, but are not required to find, that the hands of karate expert qualify as a deadly weapon under the statute." In his case-in-chief, Stimson calls Osaga, a fellow karate expert, who will testify that karate is not a dangerous sport and that the hands of a karate expert are not a deadly weapon. The prosecutor objects, arguing that rebutting evidence cannot be presented with respect to a fact that has already been judicially noticed. What ruling? You may, but are not required to find, that the hands of a karate expert qualify as a deadly weapon

Courts are split on this issue Some let the evidence be put on because it is not binding Some will not let the evidence in because the court took notice of an irrefutable fact and so the defendant should not be able to put up a case about facts that aren't to be fought over

Problem 11-L: the speed trap Danielson is clocked by police radar driving 53 mph in a 35 mph speed zone. The jurisdiction requires that before scientific evidence may be received the court must find that there is general acceptance of the technique within the relevant scientific community. The prosecutor asks the court to take judicial notice that radar has achieved general scientific acceptance as a reliable speed-measuring technique. What ruling? "I take judicial notice that radar has achieved general scientific acceptance as a reliable speed-measuring technique"

Courts will take notice that tools are reliable The procedure and method is reliable but not that the court is taking judicial notice to say the reading was accurate

Problem 13-A: a white granular substance o Swenson is arrested and three baggies containing a white granular substance are seized from his person. Arresting officer Taylor delivers the baggies to Ursula at the state crime lab for analysis. Ursula turns them over to a chemist named Vic, who runs tests and determines that they contain cocaine. Taylor picks them up from Vic on the day of trial and brings them to court. You are the prosecutor. What foundation is necessary to introduce the baggies? Must you call Taylor, Ursula, and Vic? What routine steps could these people take to make the process easier?

Need to demonstrate that the chain of custody was never compromised and only those who needed the product had it

Problem 4-A: "I got amnesia" Dustin Duran is tried on charges of racketeering and disrupting interstate commerce, arising out of the armed robbery of Halshire Foods on March 29, 2013. Before a grand jury in June 2013, Peter Breen testified that he, along with Duran and a man named Zigler, "cased" Halshire Foods several times prior to the robbery. On the night in question, Breen climbed a telephone pole and broke into the store through an upstairs window. He then let Duran and Zigler in the back door. While Zigler guarded the janitor in the office, Breen and Duran wheeled in portable acetylene tanks and a blowtorch and "blew the safe." Duran, Breen, and Zigler then fled with the proceeds. In June 2014, the government calls Breen as its star witness at trial, but he suddenly proves unhelpful. First he claims his privilege against self-incrimination. The court immunizes him from any future use of his testimony and tells him to answer the prosecutor's questions. Then he says he can't remember what happened on March 29, 2013, because he "got under the influence of Valium in May 2013 while living at the Metropolitan Correctional Center, and that made me forget." He adds that "events in the early summer of 2013 upset me because federal agents and others threatened me so I got amnesia about what happened before then, except that I remember going in the Witness Protection Program." Finally, Breen says he cannot remember making any statement to the grand jury about the robbery. The prosecutor argues that Breen's lack of memory is "feigned," hence that his position at trial is "inconsistent with his detailed grand jury testimony in June 2013." Invoking FRE 801(d)(1)(A), she offers a transcript of that testimony. Counsel for Duran objects: Your Honor, Breen's grand jury transcript is inadmissible. First, under 801(d)(1)(A), the earlier statement has to be "inconsistent" with the trial testimony, and it isn't. All Breen has said here today is that he can't remember. Second, Breen has to be "subject to cross-examination" on his prior statement, but he isn't. How can I question him on his allegations about the robbery in March of a year ago when he says he doesn't remember blowing the safe at Halshire Foods? How can I question him about what he told the grand jury last June when he can't even remember testifying? Sure, he'll sit there and let me ask questions, but that can't be all that "subject to cross-examination" means. If I can't get anything out of him, how am I supposed to test his statement? The prosecutor replies in this vein: Your Honor, Mr. Breen is a classic turncoat witness. In June 2013 he testified to the grand jury about the Halshire Foods robbery on March 29th, but now he's got cold feet. Selective memory loss, actually. He remembers going into the Protection Program, which was in May of 2013, and the defendant can ask him about that and going to the grand jury, and anything he wants about credibility. The trial court admits the grand jury testimony. On cross, Breen still claims he cannot remember the robbery, but counsel for Duran gets him to admit "I've been a burglar most of my life" until deciding "to lay low while helping the government, as long as they'll keep me in the Program and pay me." Breen admits he's been in trouble with crimes while in the Program, and "got more than $40,000 from the government over the last year, you know, for helping 'em out." Duran is convicted, and he appeals. Did the court properly admit Breen's grand jury testimony under FRE 801(d)(1)(A)? What are the best arguments on both sides? Defendant argues grand jury testimony not admissible—

Not consistent with trial testimony because trial testimony is just that witness cant remember It doesn't matter, inconsistency is anything that is different "I can't remember the underlying event" --It doesn't matter, he just needs to remember giving grand jury testimony

Problem 5-A: Fight in the Red Dog Saloon Don and Vince come to blows in a local watering hole known as the Red Dog Saloon. Both suffer serious injuries, although Vince gets the worst of it: wineglass in hand, Don takes a wild swing, and the glass shatters as it strikes Vince in the mouth, inflicting lacerations leading to permanent scars on his face. Don is charged with assault and battery. He pleads self-defense. Testimony conflicts as to who struck the first blow, though it appears that Don was seated at the bar enjoying his Chablis when Vince muttered something snide about what real men drink. In the trial of Don, the prosecutor calls Coach Jones as a witness during the state's case in chief, offering his testimony that Don is one mean aggressive physical man, quick tempered and prone to violence. Don ojbects that the proffered testimony is irrelevant and barred by the character rule. During the defense case in chief, Don calls Reverend Gram, offering his testimony that Don is peaceably disposed toward all people, gentle and nonviolent, more likely to run from a fight than to defend himself, and certainly not likely to initiate violence. The prosecutor objects that the proffered testimony is irrelevant and barred by the character rule.

Prosecutor offers testimony by Coach Jones that Don is one mean aggressive man, quick tempered and prone to violence --Not admissible because cannot admit evidence from past violence because it creates a forbidden inference Don offers his own evidence from Reverend Gram --Admissible ---First exception: can offer evidence of the defendant's pertinent trait ---Criminal defendants can offer evidence of their own character Mercy rule --Don calls Rev. Gram who offers to testify that Don is an honest person ---Not admissible because the trait must concern the crime which is about violence If court allows Rev. Gram to testify that Don is peaceful, may the prosecution call Coach Jones to offer his opinion that Don is prone to violence --Admissible because Don opened the door to put on character evidence, so the prosecutor can use contrary evidence

Problem 14-D: nine hours or one? Gretchen sues St. Anne's Hospital and Dr. Mazo for negligence in an operation that caused her permanent brain damage. At the time, Gretchen was pregnant and had been admitted for emergency surgery. While under a general anesthetic, she vomited in her oxygen mask, which blocked the flow of oxygen to her brain. After the operation she was permanently comatose, living on life support, and her baby died unborn. In his testimony, Dr. Mazo admits that it would be negligent to place Gretchen under anesthesia if she had eaten only one hour earlier. But St. Anne's introduces her original admitting record, which shows that nine hours had passed since she last ate. In a dramatic development during rebuttal, Gretchen's attorney calls Sally Abrams, the nurse who admitted Gretchen to St. Anne's. Abrams testifies that Gretchen told her she had eaten a meal just one hour prior to coming to the hospital. Abrams wrote the number "1" on the admitting form, and she testifies that after the failed surgery Dr. Mazo forced her (on threat of dismissal) to change the "1" to a "9." Before altering the form, however, she made a photocopy showing her prior entry. Gretchen offers the photocopy. The defense raises a Best Evidence objection, both to the copy and to Abrams' testimony, arguing that the copy is a forgery that does not accurately reflect the contents of the original. How should the court rule, and why? What if the judge is personally persuaded that the photocopy is a forgery?

Should the photocopy be admitted? The original's authenticity is in question so cannot use 1003 It is a jury question if this is an accurate rendition of the original (1008)

Problem 2-E: The Battered Wife Virginia died of a stab wound in her chest, and the state has charged her ex-husband Donald with murder and, in the alternative, manslaughter. Donald has pleaded not guilty to the charges, claiming that the killing was accidental. There is no doubt of the cause of death. Virginia's body was found in the trailer where she lived with Todd and Jason (her children), dead from a massive chest wound. Nor is there any doubt that Donald played a role in her death: the evidence shows that he called the police at 2 AM saying that he had just stabbed his wife. During the state's case in chief, the prosecutor proves cause of death and introduces a knife said to be the fatal weapon, along with lab analysis connecting the blood on the knife to Virginia, forensic testimony that a knife of that size made the wound, and evidence that Donald's latent fingerprints were on it. During the defense case, Donald testifies that he spent the evening at the trailer watching television with Virginia while the children were asleep, and the two fought when Virginia told him she was leaving and taking her children with her. He testifies that he asked her not to go and he would try to get a court order to stop her from leaving him. He testifies that Virginia then attacked him with a baseball bat and admits he picked up the knife from the countertop, but says she fell into the blade and he didn't even know she was hurt. During its case in rebuttal, the state offers testimony by a counselor at a shelter for battered women that two years earlier Virginia had sought refuge there for about 30 days, during which time she divorced Donald. Would evidence of a wife seeking refuge in a shelter two years earlier be admissible on murder charges if self-defense claim had not been asserted?

This would not be admitted If she had raised the self defense claim then it would have been relevant to raise the shelter because it would have shown that she has had issues in the marriage previously

Problem 4-L: Fright points the finger Donald is tried for the murder of Virginia, who is found shot in the living room of their home. A kitchen knife is found on the floor close to her body, and the gun that fired the fatal shot is recovered. There is testimony from neighbors that their relationship had been stormy, punctuated with loud fights and occasional violence. Other evidence points toward Donald's guilt, all of it is circumstantial. The prosecutor wants to offer the following proofs to bolster his case: Several weeks before her death Virginia told her neighbor, "I'm afraid Donald is going to kill me"; Days before her death Virginia told her neighbor, "I'm going to take the train to Denver to stay with mother for a while"; A few months before her death Virginia left home temporarily and took refuge in a shelter for battered women. Should these items be excluded as hearsay? Does the state-of-mind exception apply?

"I'm afraid Donald is going to kill me" --Not admissible because it invites impermissible inference about acts that caused state of mind A few months before her death Virginia took refuge in a shelter for battered women --Admit because it is non-assertive conduct --Not intending to assert to the world that she had been battered "I'm going to take the train to Denver to stay with my mother for awhile" --Being offered to prove that she is going to Denver to get away from her abusive husband it is inadmissible because it is the causation of her state of mind --if being offered just to say that she intended to go and stay with her mom and not for the reason she wants to leave it is admissible

D is prosecuted for murder of his wife. D admits that he shot his wife but his defense is that the shooting was accidental. D calls a police officer to testify that several hours after the shooting he had a conversation with D in which D stated that he was sick and grief stricken over his wife's death

Admit --Direct statement of state of mind He may be faking it and the jury may not give it much weight It is relevant about whether he did feel grief

P sues D for damages for wrongful death arising out of accident where P's husband H was killed. D calls a witness who will testify that six months before H's death H had said several times that he hated P and was very unhappy in his marriage

Admit it --Direct statement of his state of mind Jury would want to know how long would the marriage of lasted

Problem 2-F: The Exploding Gas Tank Struck from behind by a vehicle exceeding the speed limit on the highway, the car in which Risner was riding as a passenger bursts into flames as a result of a ruptured fuel tank. Within 24 hours, Risner dies from burns sustained in the accident, and his widow thereafter sues the automaker, alleging negligent design of the fuel tank caused Risner's death-that if the tank had been properly designed it would not have ruptured when the car was struck from behind. At trial, defendant automaker introduces testimony by a state trooper that the impacting vehicle was going about 80 mph at the time of the crash. The automaker also introduces a certified copy of a guilty plea, entered by the driver of the impacting vehicle to charges of involuntary manslaughter arising from the accident. In the end, the jury returns a verdict for defendant. Should defendant automaker be allowed to introduce evidence that the driver of the impacting vehicle plead guilty to charges of involuntary manslaughter arising from the accident?

Best ground for FRE 403 objection? It is confusing or misleading for the jury what the issue in the case is They may think it is the fault of the driver that hit the car rather than the automaker who developed the dangerous car Do not admit about the guilty plea because it would be misleading to the jury

Problem 11-K: Lighter fluid explosion Goodman sues Inland Chemical (Inland) for injuries suffered when a can of charcoal lighter fluid manufactured by defendant exploded. Goodman had attempted to light the charcoal in his outdoor grill, but when he checked 15 minutes later he did not see any flames or feel much heat. He did notice that some of the briquettes were grayish-white at the corners. He poured more fluid on the charcoal. Immediately, a flame shot up the stream of fluid and the can blew up in his hand. Inland asserts the defense of contributory negligence, which under the applicable law constitutes a complete defense if proven. Inland moves for a summary judgment, on the ground that Goodman was contributorily negligent as a matter of law. Inland asks the court to take judicial notice that once combustion occurs in a bed of charcoal, the addition of flammable fluid is certain to result in instantaneous flare-up of the volatile liquid coming into contact with the charcoal. Should judicial notice be taken? Is this matter adjudicative, evaluative, or legislative? In ruling on this motion for summary judgment, I am taking judicial notice that pouring lighter fluid on burning charcoal will result in an instantaneous flare-up.

Court said the issue was beyond reasonable fact regardless so they took notice anyway

Problem 11-B: the subpoena West is ordered to show cause why he should not be held in contempt for failure to appear as a witness at a trial after being served a subpoena. West denies getting the subpoena. The petitioning party asks the court to take judicial notice that West was served, directing the court's attention to a properly executed return of service filed by the sheriff and included as part of the record of the case. The petitioning party also asks the court to take judicial notice that West had been held in contempt for failure to appear at an earlier trial of the same case, and had also been held in contempt for refusing to testify in related cases in two other states. What ruling? Your honor, I request that you take judicial notice of the following fact: that West was properly served with a subpoena

Do not allow · It cannot be accurately and readily determined from sources (201(b)(2)) Based on the records of this court, I request that you take judicial notice of the following fact: that West had been held in contempt for failure to appear at an earlier trial in this same case · Allow Courts usually take notice of their own records Based on the court records in two other states, I request that you take judicial notice that West was held in contempt for refusing to testify in related cases · It would have to be accurately and readily determined to be considered

Problem 2-J: The Exploding Tire Herb Lewis installs tires and batteries for the Auto Service Center operated by Nationwide Mercantile. One day he begins to mount four snow tires on a car. He finishes three and places the fourth on the wheel rim. Inflating it, he watches the bead rise along the inner edges of the rim, waiting for the pop when it would jump and firmly seat itself in the lip of the rim. This time the bead strikes the lip and the tire explodes, sending Herb to the emergency room with serious injuries. Len Small, manager of the Service Center, gathered up the burst tire and sent it to be tested by Failsafe Automotive Laboratory. Herb sues Grather Tire Company, alleging that it made the tire and it was defective. During his case in chief, Herb seeks to establish (1) through testimony by Len Small that Grather made the exploding tire, and (2) through testimony by Michael Treaver (who tested the tire Failsafe got from Small) that the tire was defective. Unfortunately, Small did not note the markings on the tire, and Treaver failed to record them in his report, so nobody knows who made the exploding tire. Herb seeks to elicit from Small that Grather made 80% of the tires at the Service Center, and that four other manufacturers account for the remaining 20% in about equal proportion. Grather objects, arguing that the court should not permit gambling odds testimony and mere numbers cannot support a verdict. What is the problem with finding liability based on the fact that defendant made 80% of the tires sold at the store?

Doesn't that show by a preponderance of the evidence that he made this tire? Just the mathematical proof is not enough

Problem 11-F: asbestos and cancer Plaintiffs who contracted cancer after exposure to asbestos file a product liability action against a leading asbestos manufacturer. Plaintiffs request that the court take judicial notice that asbestos causes cancer. What ruling? Assume the request is granted. Is defendant then barred from trying to prove the contrary? Is defendant barred from arguing to the jury that asbestos does not cause cancer? Assume the court has taken judicial notice that asbestos causes cancer and tells the jury to find this fact

During defendant's case in chief offers an expert to testify that asbestos does not cause cancer · Not admissible The jury MUST accept the noticed fact as conclusive (civil cases)

Problem 2-A: Too Much Wax on the Floor Juliette Bryant, a 60 year old married woman, went grocery shopping at her local Alpha Market on Tuesday, January 3. The weather was unseasonably warm and dry. While pushing a cart down one of the aisles, Mrs. Bryant fell and sustained a distal fracture of the lower one third of the right fibula. Her foot and leg had to be put in a cast for six weeks, and she was hospitalized for 30 days. According to her doctor's testimony, she developed traumatic arthritis causing chronic partial disability that might be permanent. Bryant and her husband sue Alpha market, claiming that it failed to maintain its floor in a safe condition. At trial she shows that each Saturday night Alpha cleans the floor by scrubbing with water and detergents, followed by machine scrubbing to remove old wax and dirt and then the application of a solution of new wax and water. The evidence indicates that the amount of water in the solution determines how much wax adheres to the floor, and that improper mixtures lead the wax to cake and become slick. This procedure was followed on the Saturday before the mishap, and the store was closed on the following Sunday and Monday in observance of new year's holidays. Mrs. Bryant was among the earliest customers on Tuesday morning. Plaintiffs call Mr. Walters, the manager of Alpha Market, as an adverse witness. Over Alpha's objection, plaintiffs get Walters to admit that he has also slipped on the floor when it was over-waxed and received several reports of customers falling in the past year. Should plaintiffs have been permitted to ask Walters about his own prior falls? About the reports he received that others fell?

Evidential hypothesis in offering evidence of prior falls? Prior falls indicate floor was dangerous when over-waxed Defendant had notice of that danger Yes it can be admitted

Problem 5-Q: this is criminal; you cant exclude civil settlements here Michael Prince founded First American Provident Company in Indianapolis, Indiana, with the purpose of overseeing and managing services used by insurance companies. Prince approached investors and persuaded 140 of them to provide more than $2 million in capital for First American. In fact, Prince did not use the money as promised, but spent it for his own personal pleasures, eventually looting First American of all its liquid assets and profiting to the tune of more than $1.5 million. Alerted to the situation, the Securities Division of the State of Indiana investigated Prince for violations of the Indiana Blue Sky laws (statutes regulating sales of securities). The investigation led to negotiations involving Rachel Sanders, on behalf of the Attorney General, Quill, who acted as counsel for Prince, and Michael Prince himself. These discussions in turn led to a stipulated "cease and desist order," an agreement by Prince to pay a fine in the amount of $250,000, and his commitment not to solicit investments in the State of Indiana for ten years. Later the Department of Justice indicted Prince in federal court in Indiana for mail fraud in connection with First American. At trial, the federal prosecutor offers a transcript of a conversation involving Prince, Quill, and Sanders, in which Prince admitted making personal use of more than $1.5 million raised from investors. The prosecutor also offers proof that Prince paid the civil fine in Indiana. Quill, once again representing Prince, objects: "Your Honor, that's settlement talk, he can't use that. Civil settlements are covered by the Rule, let's see, it's Rule 408." The prosecutor disagrees: "This is criminal, Your Honor, you can't exclude civil settlements here." How should the court rule, and why?

IN the federal mail fraud prosecution, can prosecutor offer evidence of defendant's earlier agreement to pay? --Not admissible because this is a civil settlement in a later criminal case Criminal case but there was civil negotiation in a government agency --Admit, the trap is in language in pt. 2 --Except when offered in a criminal case and when the negotiations related to a claim by a public office in the exercise of its regulatory, investigative, or enforcement authority

Problem 13-J: the death certificate Walter Bellamy Jr. died in a single-car accident when his car went off the road while he was driving home. His life insurance company refused to pay death benefits to his widow, the designated beneficiary, claiming that his death was a suicide, hence outside the coverage of the policy. His widow files an action against the life insurance company. To prove the cause of his death at trial, she offers the certified copy of his death certificate shown below. Is it admissible as a self-authenticating document under FRE 902(4)? Why or why not? What if a second certificate were stapled to the death certificate, apparently signed by "Charles Burkett" above the title "Clerk of Cook County," bearing the apparent seal of the Office of County Clerk and containing the following statement: I certify that I am the Clerk of Cook County, and that Emily F. Bundy is personally known to me as the Deputy Clerk of Cook County, having legal custody of the records of the county pertaining to births, deaths, and motor vehicle registration. Would this certificate authenticate the death certificate? Should the death certificate be admitted?

It was signed by the custodian There is a glitch in the rules Restyled 902 says long as the copy is certified as correct by the custodian or the person Original 902 said the copies the copies have to have a seal

Problem 3-B: Kenworth and Maserati A huge Kenworth truck pulls up to an intersection regulated by traffic lights and stops in its right hand lane. An open Maserati sports car pulls up beside the truck in the left lane and stops, and Phillip behind the wheel guns his engine. The traffic light across the street is not working, and Phillip cannot see the light nearest him, above and to his right, nor can he see cross-traffic coming from his right, as the bulk of the Kenworth is in the way. The Kenworth begins to pull forward, and Phillip in the Maserati shoots into the intersection, where he is broadsided by a blue car crossing from its right, driven by Hillary. A lawsuit follows, for personal injuries and property damage, with Phillip and Hillary each claiming the other was at fault. As proof that he had the light in his favor, Phillip offers to testify that the truck pulled forward across the pedestrian lane into the intersection before he (Phillip) stepped on the accelerator. Hillary objects, her lawyer arguing that evidence of the behavior of the truck driver at the intersection is hearsay, when offered to prove that the light had turned green for Phillip. The trial judge is astonished: "Hearsay? What in the world are you thinking of, counsel, there isn't an assertion in sight here. Overruled." You are Hillary's lawyer. Advance her argument that evidence of the movement of the Kenworth is hearsay, when offered for the stated purpose. For the moment, do not try to apply Rule 801. Just explain to the judge exactly what logical steps are required to get from the movement of the truck to the conclusion that the light was green for Phillip, and why those steps involve the same risks that lead us to exclude an out-of-court statement ("the light's turned green"), when offered to prove what it asserts. If you can't do it, read the next case and try it again. Testimony is offered that the truck started to move forward at the intersection

Offered to prove light was green In common law, this is viewed as hearsay Two step inference --Conduct of the actor will suggest a belief of the actor --Staring at intersection indicates truck driver believed light was green Plaintiff offers truck driver's belief as evidence at trial to prove the truth of that belief, that in fact the light was green

Problem 4-H: I was on an errand for my boss Driving a truck bearing the legend "Farmright Produce Corp.," Rogers collides with an automobile driven by Story. Neither is badly hurt, but the vehicles (especially the car) are damaged. Some 30 minutes after the accident, Rogers remarks to Story, "I'm sorry this happened. I was making a delivery for Farmright, and got distracted for a moment trying to read the purchase order on my clipboard." Story sues Farmright Corp., which includes in its answer to the complaint an averment that "the driver Rogers was not acting within the scope of his employment at the time of the alleged accident." At trial, Story offers to testify to what Rogers said, invoking FRE 801(d)(2)(D). Farmright objects: Your Honor, that exception cannot be invoked unless plaintiff proves that Rogers was an agent of Farmright speaking of a matter within the scope of his employment. Plaintiff has not proved either point, and we are prepared to go forward with evidence that he was neither. In short, the necessary foundation for using that provision has not been laid. Moreover, Your Honor, they can't use the statement to prove the very facts which the Rule sets out as conditions of admissibility. That would be bootstrapping. What result, and why? Check FRE 104(a).

P wants to offer an incriminating statement made by truck driver Driver says two things --I was making a delivery for Farmright --I got distracted reading my clipboard Who decides whether the driver was acting within the scope of his employment in making the statement - the court or the jury? --the judge In deciding whether employee made statement while acting within the scope of employment can the court consider the statement itself? --the court can consider the statement but it also needs other facts on which the admissibility depends Bootstrapping --The court can consider the statement "I was on an errand" but it cannot consider the statement "I got distracted" because it does not establish more

Problem 3-F: "I'm from the gas company" While working on the job at the Crane Wrecking Company, Jack Alford thought he smelled the odor of natural gas coming from a nearby pipeline running above ground. Although Alford had not yet decided to call the gas company, a man appeared on the scene shortly thereafter: "I'm Joe Forrest from Interstate Gas," he said, "Could you show me where the pipeline is, so I can check it out? We've had reports of a leak in this area." Alford took Forrest around to the back of the building and was pointing out the pipeline when Forrest started to light up a cigarette. The burning match ignited the escaping gas, and in the explosion Alford was seriously hurt. Alford sues Interstate Gas on a negligence theory, and Interstate raises contributory negligence as a defense, arguing that Alford should not have gone so close to what he knew or suspected to be a gas leak. At trial, Alford offers to testify to what Forrest told him just before the explosion, as proof that Forrest was an agent of Interstate Gas and that Alford's behavior was reasonable. Interstate Gas raises a hearsay objection. When offered for either or both of these purposes, is Forrest's statement hearsay?

Plaintiff Alford offers to testify that Forrest told him "I'm Joe Forrest from Interstate Gas" offered to prove that Forrest worked for Interstate gas --Do not admit because it would be to prove the truth of the matter asserted Plaintiff Alford offers to testify that Forrest told him "I'm Joe Forrest from Interstate Gas. Can you show me where the leak is so I can fix it" and Offered to prove that Alford wasn't contributorily negligent in getting near the leak --Admit because it is used to prove that the behavior was reasonable Proof that someone from the gas company told him to show him where the leak was --Show the effect on declarant's state of mind and why he would have been standing so close to it

Problem 14-A: the defamatory letter Paula brings a defamation action against Daniel based on statements made by Daniel in a letter to Paula's employer. At trial, the letter is neither produced nor shown to be unavailable. Over a Best Evidence objection, the employer is allowed to testify regarding the contents of the letter. Paula is awarded a substantial verdict. On appeal, Daniel contends that it was a clear violation of the Best Evidence rule to allow the employer to testify to the contents of the letter without being required to produce it. Is he right? Did Paula need to prove the "content" of the letter to win her defamation action? Assuming the appellate court agrees with Daniel, is the error likely to be found to be harmful? Should the judgment be reversed and the case retried? To resolve this question, what question is Daniel likely to be asked at oral argument?

Should the case be reversed and retried? Would need to know if the testimony represents the substance of the letter

Problem 5-P: Two potato, one potato Potato farmer Amos Perrin purchased from salesman Evan Sosbee an herbicide called Perquod made by Cheron Chemical Company, to apply to his crop. Sosbee was an agronomist, and his job was not only to sell Cheron products, but to deal with customers by offering suggestions on application and addressing complaints. Appearing at the Perrin farm at the initial application of the Perquod, Sosbee advises Perrin that "Cheron will back up its recommendation of the product." Midway through the growing season, Sosbee again appears on the farm, and Perrin comments that "the stuff we put on killed the weeds alright, but my potato plants don't look as healthy as they should." Sosbee replies, "Don't worry. We'll take care of you." At harvest time, Perrin again sees Sosbee, and Perrin, now clearly unhappy, advises Sosbee that "my yield isn't half what it was last year." Sosbee replies, "You just tell us the damages you're claiming, and we'll bill Cheron." Cheron doesn't pay. Perrin sues Cheron. At trial, he offers to testify to the various statements by Sosbee. Assume that his comments would normally be admissible against his employer, but Cheron objects that his statements were "offers to settle Perrin's claim, and as such they are excludable under FRE 408." How should the court rule and why?

Admissible, there is no dispute here so 408 is not triggered They were admissions of liability

Problem 3-C Abby sues Burton for property damages and personal injuries arising out of an intersection collision. (Abby had been driving a maroon station wagon, Burton a blue sedan.) In order to show that Burton's blue sedan ran the light, Abby's counsel calls Bystander and establishes through preliminary questions that he saw the accident. Then the critical question: "Which car had the light in its favor?" Bystander's response: "The light was green for the station wagon. The blue car ran a red light." On cross-examination, Burton asks Bystander about a conversation he had with insurance adjuster Charles three days after the accident. Over Abby's hearsay objection, Burton wants to ask Bystander whether he said to Charles, "The blue car had the green light in its favor." Burton replies to the objection, "Your Honor, we seek only to impeach the witness, not to offer the statement for its truth." "Didn't you tell the insurance adjuster that the blue car had a green light" Offered to impeach the witness's testimony that the blue car had a red light

Admit, it is not hearsay because it is not for the offer of proof but for impeachment

Problem 14-B: the un-private physician o Denise retains Dr. Murphy to arrange a private adoption of her newborn daughter with an express understanding that her identity as mother will never be disclosed to the adoptive parents or the child. Eighteen years later the daughter seeks her biological parents and is put in touch with Dr. Murphy. At her request, he gives her a photocopy of his adoption records, which identify Denise as the mother. The daughter contacts Denise, which results in psychological anguish for Denise. She sues Dr. Murphy for outrageous conduct and breach of promise. At trial, Denise offers the photocopy of the adoption records that Dr. Murphy gave her daughter. His lawyer objects, saying the Best Evidence rule requires production of the original records, not the photocopy. What ruling? Would it matter whether the jurisdiction had adopted Rule 1003?

At trial the plaintiff offers the photocopy of the adoption records that Dr. Murphy had given to her daughter the photocopy would be considered the original so it can be admitted 1001 says an original can be any counterpart intended to have the same effect by the person who executed or issued it

Problem 14-G: the recorded conversation In a drug surveillance operation, DEA agent Nolan monitors a conversation between Peter and Quinn that takes place in a room in the Quality Court Motel. Using a planted bug and recording equipment in an adjacent room, Nolan both hears and records what Peter and Quinn are saying. The conversations relate to drug transactions, past and ongoing. After arresting Peter and Quinn, Nolan interrogates them separately at DEA headquarters. First Nolan questions Peter, who admits his involvement in a drug scheme and implicates Quinn. After getting Peter's story, Nolan summons a stenographer and has Peter repeat his statement. The stenographer prints out a hardcopy, and Peter signs it. Then Nolan questions Quinn and tape records the interview. Consider the following evidence issues: At the trial of Peter and Quinn, should Nolan be allowed to testify to the conversation he overheard at the Quality Court Motel or does the Best Evidence doctrine require the government to produce the recording? Can Nolan testify to Peter's admissions about his own conduct, or does a Best Evidence objection by Peter force the government to offer the signed written statement instead? What if Nolan thinks the signed statement omits important points that Peter uttered in his original recitation, before the retelling that the stenographer used in making the written version? Can Nolan testify to what Quinn said at DEA headquarters, or does a Best Evidence objection force the government to offer the recording?

Can Nolan testify to the conversation he overheard in the motel room? · The conversation was recorded · This is admissible · He is testifying to what he was hearing because he was present that the time of the incident · He can testify to what was actually said regardless if the tape reorder was on because he heard what was happening Can Nolan testify to Peter's admission about his own conduct? Admission was recorded Can Nolan testify to the confession Quinn made at DEA headquarters? Confession was recorded by a stenographer

Problem 4-Q: If you want to stay healthy Lanny Keeton is charged with armed robbery of Southside Quick Serve in St. Louis, and with assault and attempted murder. The crimes occurred at 2 a.m. when two masked men entered the store carrying sawed-off shotguns. The videotape shows that the two trained their guns on Nick Owens, the night-duty clerk in the cashier's cage behind bulletproof glass. Owens turned over the cash (as he was required to do in such situations), but the amount was small (less than $100) because he had just put most of the cash from the register into the safe, which could only be opened by dialing the combination and waiting ten minutes. One perpetrator is seen on the videotape menacing a customer with the shotgun, and the customer is forced to lie face down on the floor. The videotape shows this perpetrator firing his shotgun through the change slot that scoops below the glass in the counter, injuring Owens who is struck by pellets that ricochet up inside the cage. Ten days later, armed with minimal information supplied by the customer and Owens describing the getaway car, police arrest Marvin Spreigel in a routine traffic stop. (A search of the Police Information Network alerted the patrolman that Spreigel and his car might have been involved in the Quick Serve robbery.) Later that same day, police also locate and arrest Lanny Keeton, and both wind up in the same station house, where they are questioned separately. Spreigel acknowledges his participation in the robbery and says Keeton fired the shotgun at Owens and threatened the customer. The prosecutor enters a plea bargain with Spreigel, who pleads guilty to aiding and abetting and agrees to testify against Keeton. When Keeton's case comes to trial, however, Spreigel refuses to testify. In a hearing in chambers attended by Spreigel and his lawyer, from which Keeton and his lawyer were excluded, Spreigel's lawyer reads a letter that he says Spreigel received from Keeton in jail while awaiting his sentencing hearing after entering his plea. The letter says "if you and that bitch of yours want to stay healthy, you know what you should do and what you shouldn't, so I better not see you as a stoolie at my trial." Spreigel's lawyer tells the judge that Lanny Keeton made oral threats targeting both Spreigel and his girlfriend. The judge rules that statements Spreigel gave to police and prosecutors, as well as his later guilty plea allocutions, are admissible against Keeton under Rule 804(b)(6). Informed by the judge that the ruling rested on "conversations with the prosecutor and Mr. Spreigel and his lawyer in chambers," the lawyer for Keeton raises a hearsay objection, and argues that "you can't find my client responsible for Spreigel's refusal to testify in an ex parte hearing on the basis of unsworn statements by a lawyer," and "besides, any finding that my client kept Spreigel off the stand must rest on proof beyond a reasonable doubt." Do these objections have merit?

Can declarant come to the hearing? --Yes they can come Can defendant come to the hearing? --No, only the attorney can come What evidence can come in at the 104(a) hearing? --Don't have to follow the rules of evidence? What is the standard of proof at the 104(a) hearing? --Preponderance of the evidence

Problem 5-K: Acting out on the assembly line Rita Haines works on the assembly line at Danmore Tractor Company. Most of the people she works with are men, although the number of women on the line is now approaching one in five. In federal court, Haines sues Danmore and her supervisor Sam Torgenson, alleging sexual harassment. (Federal law affords relief against employers and others in the work force for sexual harassment, which can include hostile work environment. Whether employers are liable is affected by steps that they take—or don't take—in addressing problems through policies, enforcement, and response to complaints. To prevail, claimant must show that she personally found the conduct hostile, abusive, or offensive, and that a reasonable person in her position would react that way.) Haines alleges that she was regularly forced to endure unwanted sexual innuendo directed at her personally, that fellow-workers made comments about her appearance and figure, that Torgenson propositioned her in the presence of others, that she had to listen to stories about sexual encounters and adventures, and that pornography sometimes appeared in the lunchroom. At trial, Danmore offers proof that (1) on weekends Rita works occasionally as an exotic dancer in a local club, (2) she sometimes reports to work in a mini-skirt and a tight blouse (although she changes to work clothes before starting her shift on the line), (3) she told at least one sexually explicit story in the lunchroom that apparently rested on some kind of personal experience, and (4) Rita made sexually suggestive remarks on several occasions to two of the men on the line. Haines' lawyer objects to all of these items of proof, invoking FRE 412. How should the court rule, and why?

Danmore offers evidence that Rita works as an exotic dancer on weekends and that she often reports to work in a short skirt and tight blouse and then changes into her work clothes --Not admissible because She has the right to work as an exotic dancer --Would have been unwelcome conduct because her behavior was done outside of the workplace Offers evidence that she made explicit comments while at work Admit because if she is telling provocative stories it is harder to say that it was unwelcome --Rebuts the claim that it was totally unwelcome

Problem 5-B: Red Dog Saloon Part 2 In the trial of Don for the assault on Vince, Don calls Ernie, offering his testimony that Vince is a belligerent, fight picking, aggressive fellow with a real short fuse. The prosecutor object that the proffered testimony is irrelevant and barred by the rule against character evidence.

Don calls Ernie to testify that Vince has a short fuse --Admit ---Rebuttable evidence can be admissible ---Can offer evidence of a defendant's character Could prosecutor offer evidence that the victim had a peaceable character? --Yes there is a special provision for homicide cases --Prosecutor can be the first one to raise character evidence (FRE 404(2)(c)) Assume the court allows Ernie's testimony that Vince is a belligerent, fight picking, aggressive fellow, may the prosecutor call witnesses to testify to defendant's violent character? --If a defendant offers evidence of an alleged victim's pertinent trait, the prosecutor may offer evidence to the defendant's same trait ---So yes, this is admissible If this were a homicide case and D put on evidence that V was the first aggressor, but didn't offer any character evidence about the victim, could the prosecutor offer evidence that the victim had a peaceable character? --FRE 404(2)(c) ---In a homicide case, the prosecutor may offer evidence of the alleged victim's trait of peacefulness to rebut evidence that the victim was the first aggressor Assume the court lets Ernie testify and offer his opinion that Vince is aggressive and belligerent · May Ernie explain that opinion by saying he has seen Vince get drunk and start a fight at bars at least three times in the last year? --FRE 405: methods of proving character ---When evidence of a person's character or character trait is admissible, it may be proved by testimony about the person's reputation or by testimony in the form of an opinion and can only bring up specific instances on cross examination

Problem 10-B: the death of Mason Parnell Mason Parnell was a wheat farmer. At age 49, he died of a head wound inflicted by discharge of a 30.06 rifle while alone in the spare sleeping room in the basement of his house. His widow Vera Parnell sues Midcontinent Casualty Company on an accidental death policy covering her husband, seeking recovery in the amount of $100,000. Midcontinent claims that suicide was the cause of death and denies all liability. In such cases claimant bears the burden of pleading that death came by accident, and also the burdens of production and persuasion on that point. Physical evidence shows that Mason Parnell was flat on his back on the bed when the gun discharged and that the muzzle was close to his chin at the time. There is other circumstantial evidence. Vera Parnell introduces testimony that (1) Mason Parnell did not shoot himself intentionally because no soot pattern or flash burn was found on his face (indicating that the muzzle was more than 12 inches away), (2) a rifle owned by Parnell accidentally discharged during an earlier hunting trip, (3) Parnell died clutching a cigarette lighter (hence he may have had only one hand free to hold the rifle and pull the trigger), and (4) Parnell was in good financial condition, healthy, happily married, and not moody or morose. But Midcontinent also introduces evidence that (a) the rifle that caused Parnell's death was in perfect order and was not the one that had accidentally discharged earlier, (b) some nasal hairs were found on the front gunsight, and (c) Parnell had experienced marital difficulties and may have been suffering from Alzheimer's disease. At the close of the evidence, Vera requests that the jury be instructed on the presumption, arising from proof of sudden violent death, that death resulted from accident rather than suicide. Midcontinent argues that no such instruction is proper under the circumstances and moves for a directed verdict. The jurisdiction has adopted Rule 301, in the form enacted by Congress. What should the court do, and why? If a death is sudden and violent, there is a presumption that it was the result of an accident "Ladies and gentlemen of the jury, I instruct you that if you find the death in this case was sudden and violent you should presume that it was the result of an accident."

Do not give this presumption · This is a challenge of the presumed fact Presumption goes out of the case · 301 could give an inference instruction "if you find the death was sudden and violent, you may find the death was the result of an accident Reformist review would shift the burden of persuasion "I instruct you that if you find the death was sudden and violent, the burden is on the defendant insurance company to persuade you that it was not the result of an accident"

Problem 11-D: the football fan Rogers is charged with armed robbery. He asserts an alibi defense, claiming that he was watching a live broadcast of a professional football game on television with friends at the time of the robbery. To rebut this defense, the prosecutor supplies the court with a copy of television listings for the date in question and asks the court to take judicial notice that there was no football game of any type being televised live at the time of the robbery. What ruling? Your honor, I request that you take judicial notice of the following fact: there was no football game of any kind being televised at the time of the robbery

If there are records of all the stations then it can likely be admitted

Problem 2-C: Boys on the Bridge While driving under an overpass, Arthur is injured by a chunk of concrete crashing through his windshield from above. Buildright Construction Company is in the process of rebuilding the overpass, which entails removing concrete walk and railing. Arthur sues Buildright, and shows at trial that defendant put up a bulkhead to confine the fall of debris, and in the process broke away some concrete in order to insert steel beams for its support. No direct evidence shows what caused the piece that struck Arthur to fall from the overpass, but Buildright offers testimony by Carla that she saw four or five boys from junior high about four blocks from the bridge running away from the scene. Arthur objects. Should evidence be admitted that Carla saw the boys running in a direction taking them away from the scene at the time of the accident?

In the actual case it was admitted It is subjective and there can be marginal relevance about how a testimony can affect the case Stretches the outer boundaries of relevance

Problem 10-D: Presence of a firearm College student Sam Alden returns home to Rochester, New York, for a Christmas visit with his family. During his visit he encounters his former high school classmate Burnell, and late one evening the two drive around town in Burnell's car. Police stop them, and it turns out that the car matches the description of a getaway car used in a nearby liquor store holdup in which the salesperson was killed 20 minutes earlier. The arresting officers direct Burnell to open the trunk, where they discover a sawed-off shotgun and three Uzi automatic rifles. Alden and Burnell are charged with unlawful possession of firearms. At trial, the prosecutor offers evidence of the facts outlined above. He also calls Clayton as a witness, and he testifies that he and Burnell belong to a gun club and that Burnell once bragged that "I have a bunch of Uzis, and a modified shotgun." At the close of the evidence, the prosecutor requests that the court instruct the jury as follows: The presence in a car of a loaded firearm is presumptive evidence of unlawful possession. On proof of the presence of guns in the car, you may infer that each defendant in the car possessed them. The presumption is effective only so long as there is no substantial evidence contradicting that conclusion, and it disappears when contradictory evidence is adduced. To establish unlawful possession of the weapon, the prosecutor relied on the presumption as well as testimony. Accordingly, you would be warranted in returning a verdict of guilty if you find beyond a reasonable doubt that possession is proved, and an element of such proof is the presumption of illegal possession. Alden and Burnell both object, claiming the instruction violates due process. The prosecutor replies that the Supreme Court in Allen has approved such an instruction. On these facts, is the instruction proper as against Alden? As against Burnell?

Instruction requested against Burnell Allowed because he is the owner of the car, belongs to a gun club, and admitted he owns guns Instruction requested against Alden Do not allow because It was not his car so he may not know what was in the trunk

Problem 4-F: Did you rob that bank? Ivers is charged with armed bank robbery. At his trial, the prosecutor offers testimony by his friend Jessup that several days after the crime Jessup heard a conversation between Ivers and another friend named Kerwin. Consider applying the tacit admissions doctrine to these two versions of the exchange, and think specifically about the role of the trial judge. Here is the first version: Kerwin: Are you the one who stuck up First Seacoast Bank the other day? Ivers: Will you please leave me alone. Here is the second version: Kerwin: You're the one who robbed First Seacoast Bank, aren't you? It just so happens that I was in the bank when the fellow came in, and it was you, wasn't it? Ivers: Will you please leave me alone. What role should the judge perform, in deciding whether to admit Kerwin's statement and Ivers' reply? One possibility is that she could treat the matter as an issue of admissibility under FRE 104(a) and decide for herself whether Ivers' response ("leave me alone") acquiesces in what the question suggests. If so, Ivers has adopted what Kerwin suggests and the evidence is admissible against Ivers. If not, Ivers has not adopted what Kerwin suggests and the evidence is excluded. The other possibility is that the judge could treat the question as one of conditional relevancy under FRE 104(b): Under this approach, the judge would admit the exchange between Kerwin and Ivers if the judge believes a reasonable jury could conclude that the response of Ivers ("leave me alone") acquiesces in the implications of Kerwin's question, or exclude the exchange if the judge believes that a reasonable jury could not conclude that Ivers adopted what Kerwin suggested. Which approach is better? Is adopted a FRE 104(a) or 104(b)?

Jury decides (104(b)) questions of adoption of fault typically

Problem 5-I: I didn't know they were stolen After his arrest for possession of 100 stolen iPad minis (market value of about $40,000), Huddleston is charged with knowingly receiving stolen property. At trial, the evidence shows that he was trying to sell them at less than cost, but he denies knowing that they were stolen. To prove that Huddleston did know they were stolen, the prosecutor offers testimony to the effect that he had dealt in stolen property before. The owner of a store is prepared to testify that he recently sold him 30 Apple Yosemite computers for $200 apiece (well below cost). Huddleston objects that this testimony is inadmissible character evidence that should be excluded under FRE 403 and 404(b)(1).Huddleston raises two additional points. First, he asserts that the court should decide in the first instance under FRE 104(a) whether the Yosemite computers were stolen and that Huddleston knew it. Second, he argues that the prosecutor must prove these points by clear and convincing evidence. In response, the prosecutor argues that the question whether Huddleston sold them to the store owner is for the jury to decide and that the preponderance standard applies. Who is right on these points, and why?

Preliminary question of fact about if he knew the prior products were stolen (Huddleston) --Determined under 104(b)b: the jury decides whether the defendant knew about the prior stolen property Could a reasonable jury find that the defendant could have known the property was stolen --Not very protective of defendants --Judge decides whether there is sufficient evidence to support a jury finding This is not a decision that is binding on the states, the judge has to decide if the prior bad act was committed before it was sent to determination by jury Defendant argues evidence of prior bad acts is not admissible unless judge finds under FRE 104(a) that defendant knew the computers were stolen

Problem 11-A: dry pavement Paulsen sues Davis for injuries sustained in an automobile accident at an intersection in Indianapolis on September 8, 2018. Paulsen asks the court to take judicial notice that it did not rain in Indianapolis on that date and that the pavement at the intersection was dry. In support of this request, Paulsen furnishes the court with a copy of the official weather bureau record for Indianapolis on September 8, 2018, which clearly indicates that no precipitation was recorded. Davis objects on grounds that the record is hearsay and has not been properly authenticated. Should the request for judicial notice be granted, in whole or in part? Is FRE 104(a) applicable? Your honor, we request that you take judicial notice of the following fact: that it did not rain in Indianapolis on Sept. 8, 2010

Allow it so long as the weather reports of that day can be provided 201(c)(2) must take judicial notice if the party requests it and the court is supplied with the necessary information Your honor, I request that you take judicial notice of the following fact: the pavement at the intersection was dry at the time of the collision No, there is a reasonable dispute about this issue

Problem 11-1: obscene books Roost, an adult bookstore owner, is charged with possession of obscene material. At trial, the books in question are admitted. The court concludes as a matter of law that they are obscene within the meaning of the statute and so instructs the jury. The jury is left with only the question of whether the defendant possessed the books. Has the court erred? Is this case distinguishable from Gould? "Ladies and gentlemen of the jury, I am taking judicial notice of the following fact and instruct you to find that the books possessed by defendant are obscene under the applicable statute."

No?

Problem 4-G: couldn't he see the boy? Eleven-year-old Albert Garment gets out of a school bus in front of the farm where he lives with his parents. He must cross to the other side of the highway to get home, so he walks around the front of the bus. In the meantime busdriver Martin Grider checks the traffic and, unaware that Albert is crossing in front, pulls forward. As the bus enters the traffic lane it runs over and kills Albert. The parents of Albert Garment bring a wrongful death action against Martin Grider on a negligence theory. Shortly before the statute of limitations was to run, the Garments file an amended complaint that includes a second count stating a claim in strict liability against Standard Bus Sales. The new count alleges that Standard sold the bus to the School District and that the mirrors "were so positioned that a complete view of the area within the path of the bus was not discernible by a person in the driver's seat." But the Garments have named the wrong seller, and before trial Standard wins summary judgment on the ground that it did not sell the bus. The case goes to trial against Martin Grider alone. Invoking FRE 801(d)(2)(C), Grider's lawyer seeks and obtains permission to read to the jury the allegations about the mirrors appearing in the now-dismissed second count. (Counsel for the Garments is also permitted to read Standard's denials of those allegations.) The jury returns a verdict for bus driver Martin Grider, and the Garments appeal, arguing that the trial court should not have permitted the superseded pleading to be read into evidence. Do they have a good argument? In defending the action of the trial judge, shouldn't Grider argue on appeal that the Garments' pleading entitled him to a directed verdict? Is the prior pleading admissible over a hearsay objection?

The attorney sent a letter to the bus manufacturer saying they were negligent in their design and so the bus driver wants to use it because it shows they were not negligent This can be permitted because the attorney is an authorized agent Any other ground for exclusion --403 because admission is confusing the issues or misleading the jury

Problem 3-D: "Any way you like" The state seeks an injunction to close down the Gentleman's Massage Parlor, on ground that its owner Ratliff operates it for purposes of soliciting prostitution. The government calls undercover agent Wallis to testify that while posing as a patron there he was served by a masseuse named Debra who, in the middle of giving him a rubdown, asked whether I was "interested in a good time." Wallis testifies that he replied "that depends on when and where and how much" and that Debra replied, "the cost depends on what you want, but I'm real versatile like, and you can have it any way you like, honey." Undercover agent Wallis offers to testify that masseuse told him "You can have it any way you like"

This can be admitted Do not have to prove the person would have followed through with the solicitation because any words or conduct that have independent legal significance can create legal liability

Problem 3-J: My Husband is in Denver In FBI interviews, eyewitnesses identify Greg Hensen as one of the men who robbed Girard Bank & Trust in Boston on April 10th. On April 11th, FBI agents get an arrest warrant and go to Greg's house, where they encounter his wife Barbara. "My husband is in Denver," she tells them, "because his mother just died, and he flew out to her funeral on the 9th. He's coming back day after tomorrow." On checking airline passenger and reservation lists, the FBI learn that there is no record of a Greg Hensen flying either into or out of Boston in the previous seven days, nor any reservation in his name on any incoming flights to Boston in the next three days. On April 12, agents question another suspect, who tells them that "Greg is hiding out at his brother's in Quincy." On the basis of that tip, the agents promptly get a warrant authorizing them to search for Greg in his brother's apartment, and there they find him. At Greg's trial in federal court for armed bank robbery, the government offers testimony by the agents describing their encounter with Barbara, quoting what she told them of her husband's whereabouts. Counsel for Greg objects: "Barbara Hensen is not on trial here, her husband is. What she said or thought is hearsay, and irrelevant besides." Is Barbara's statement relevant? Is it hearsay? Government offers to prove wife thought husband was guilty

Wife lies so it is not being offered for truth --It will be admitted Two step inference in common law --Wife engages in conduct of lying, which shows her belief her husband is guilty --Offered to prove husband is guilty

Problem 11-E: delayed shipment Pellum Construction Company of Los Angeles was interested in bidding on a project to construct a new hospital in Long Beach. Pellum hired a New York architectural firm to draft a design proposal. All bids, accompanied by the proposed architectural design, were required to be submitted to the Long Beach hospital board by September 15, 2001. Pellum hired Air Courier Services to fly the design proposal from New York to Los Angles on September 11. The design proposal failed to arrive in time, and Pellum lost the contract to another construction company. Pellum sues Air Courier Services Inc. for failing to deliver the architectural design proposal. Air Courier defends on grounds of impossibility of performance. At trial, Air Courier asks the judge to take judicial notice that there was a terrorist attack on New York City that occurred on September 11, 2001, and that conditions following that attack made it impossible to get shipment to Los Angeles by September 15th. Should the court take judicial notice of these points?

Your honor, I request that you take judicial notice of the following fact: there was a terrorist attack on NYC on Sept. 11, 2001 Courts will take notice of historical fact Your honor, I request that you take judicial notice of the following fact: the conditions following the terrorist attack of Sept. 11, including the shut down of air travel, would make it impossible to get a shipment from NYC to LA by sept. 15 Not admissible because Judge cannot say there is no other possible way to get a package there

Problem 13-I: the rejected easement Byron sues Casey, alleging trespass on Greenacre. Casey claims to be entitled to drive on a dirt road over the parcel. At trial, Casey offers a document that purports to be an acknowledged grant of easement covering the road in question given to her by Arthur, who had later given a quitclaim deed for Greenacre to Byron. There is evidence that Arthur mentioned "some easement on the property" to Byron, and all agree that Casey has a good defense if indeed she holds the easement.Casey offers the document without foundation testimony, and Byron offers no proof at all on the subject. At the end of trial, Casey seeks a directed verdict, arguing that the jury "cannot reject this easement because the document is acknowledged and self-authenticating under FRE 902(8)." She seeks an instruction "telling the jury that they must accept this document for what it is, proof of the easement."The court rejects Casey's motion and refuses the instruction. In his close, Byron argues that the purported easement is "obviously faked" and asks the jury to "pay no attention to that trumped up document." The jury returns a verdict for Byron. Should Casey have gotten her directed verdict or the requested instruction? Should Casey's request for an instruction telling the jury to find an easement be granted, when she has submitted an acknowledged easement and there has been no rebutting evidence? Does 902 create a presumption requiring the judge to instruct the jury to find a valid easement?

902 is not a presumption so it does not require a direct verdict

Problem 8-A: the hired gun In a product liability suit arising from a car accident, defendant General Motors has called Dr. Norbert Riley as an expert witness. Riley is a professor of engineering design, holding a Ph.D. and an endowed chair at the University of Michigan. General Motors intends to elicit Riley's testimony that the accident could not have happened in the manner alleged because the design of the automatic shift mechanism made it impossible for the car to go into reverse unattended. On direct examination, counsel for the defendant broaches the subject of the fee arrangement between General Motors and Riley: Q [defense counsel]: Now Professor Riley, of course you're here because General Motors has paid for your assistance in this case. Will you please advise the jury how you are paid? A [Riley]: Our arrangement, I believe, is that I will receive $1600 per day for my appearances here in court. Q: Thank you, sir. Now Professor Riley, you examined the vehicle involved in the accident, did you not? o Ensuing questioning goes to substance, and no further mention is made of the fee arrangements. On cross-examination, plaintiff's counsel raises the subject anew: Q [plaintiff's counsel]: Professor Riley, you mentioned that you are being paid here today. $1600 a day, is that right? A [Riley]: That's correct. Q: All right, sir, now could you tell us please how much you expect to be paid for your work on this case in total? [Defense counsel]: Your Honor, we have nothing to hide here, but plaintiff's counsel is trying to browbeat this witness. The professor has said how much he gets paid, and it's a lot because he's a highly trained expert. There's no need to go into dollars and cents here. It wastes time and distracts everybody from what's really at stake here. [Plaintiff's counsel]: Your Honor, the jury should know how much this man expects to get paid. I want them to know some other things too, including (1) how much he made testifying for GM last year, (2) whether he expects to testify for GM again, (3) how much he made, all told, last year testifying for automakers, and (4) approximately what proportion of his total income comes from such appearances. Q [defense counsel]: Well, Your Honor, I have to object to that outburst. He's grandstanding here, and it's settled that inquiries of the sort he proposes are collateral. I'm asking you now to advise the jury not to consider the implications of those last remarks as evidence and to understand that GM is paying this man $1600 a day, just as we said in the beginning. Can witness be asked?

A witness can be asked, but they are supposed to be neutral and should say they are not being paid for their testimony but for their time

Problem 4-C: fire in the warehouse Martin left his truck at Carter's Automotive Repair and Refinishing in Tupelo, Mississippi, because one of the brackets securing the gas tank had broken. Carter himself was in Oxford, Mississippi, at the time. The shop has separate areas for repair and maintenance, for body work, and for painting and refinishing. While the truck was being repaired, employee Dugan was working with a welding torch on a wrecked car, within five feet of the door to the paint storage room. Placing the flaming torch on the ground, he went around the side of the building to get a Coke from a vending machine. Moments later the fire alarm sounded, and Dugan returned to discover the paint shed burning out of control. The fire consumed the premises and destroyed Martin's truck. Martin sues Carter to recover for loss of the truck. As proof that Dugan's negligence started the fire (hence that Carter is liable by respondeat superior), Martin calls an insurance adjuster named Esher. It turns out that Carter spoke to Esher after the fire in the course of advancing his insurance claim for loss of the building. If permitted, Esher will testify that Carter told him, "The fire started in the paint shed when Dugan put a flaming welding torch on the ground too close to the fumes." Carter raises a hearsay objection, and Martin invokes the admissions doctrine and FRE 801(d)(2)(A). Should Carter's statement come in? Quite apart from what the Rule indicates, what is the sound result here? The statement is offered to the proof of the matter asserted (the cause of the fire

Admissible Statement of opposing party does not require personal knowledge

Problem 3-K: King Air YC-437 Bruno and others are charged with theft of an airplane and with importation and possession of marijuana. The evidence indicates that the plane in question, a King Air bearing identification Number YC-437-CP, was stolen in Florida and flown to an airstrip on Bruno's rural Arkansas property. There federal agents attached a transponder enabling them to track the plane when Mason and Pell flew it first to Acapulco (where it was seen picking up marijuana) and then to Mississippi (where the cargo was unloaded). Arrests were made and charges brought against Bruno and three others. Pell was not charged, for he bargained a plea and testified for the government. At his trial, Bruno contends that his involvement in the unsavory theft and drug scheme was completely innocent. He testifies that the King Air made an emergency landing at his airstrip, where he let it stay until it could be repaired, that mechanics came thereafter and fixed a ruptured oil line that forced the landing, and that the plane "was mysteriously flown away about a week later." He denies that he accepted money from his codefendants in exchange for letting the plane stay, and insists he knows nothing about the theft or use of the plane to import marijuana. Bruno also calls Kay Dixon as a witness and offers her testimony that "Bruno told me in front of six other people that he was storing a King Air at his airstrip." He argues that the fact that he said in public that he was storing the plane supports his claim of innocence, since "a man with guilty knowledge is not likely to advertise his possession of stolen property." The prosecutor raises a hearsay objection. What result, and why? "I am storing a King Air at my airstrip"

Admit It is not necessarily for the proof of the fact, the out of court statements are to support his claim of innocence to prove his state of mind He must have thought there was nothing incriminating about that and that is why he talked about it Showed his state of mind of innocence

Problem 5-N: The Burning Sofa Evan Girard co-owned and shared a house with Matt Rollins that caught fire and burned. The blaze took the life of Rollins, whose body was found in the laundry room near the back door. Rollins had been a successful developer of computer software, and had recently sold patent rights for more than $2 million. Girard sues the Rollins Estate to recover the value of Girard's interest in the house and the value of lost possessions. An expert fire investigator testified that the blaze started in the den. The investigator was sure that the origin was not an electrical short, but he could not pinpoint the cause. Another expert testified on the basis of an autopsy that Rollins had a blood alcohol content of .15, which would have caused significant physical and mental impairment. Girard knows that Rollins was a binge drinker who, when intoxicated, tended to smoke and fall asleep. Girard's theory is that the fire started when Rollins drank too much, lit a cigarette, and passed out on the sofa in the den. The untended cigarette caught the sofa on fire, which led to the blaze that damaged the house. Rollins was trying to get out the back door when he succumbed to the smoke and later died from smoke inhalation. Girard himself wants to testify to Rollins' drinking and smoking. If allowed, he would testify that Rollins was an episodic binge drinker who was usually functional, but would sometimes drink and smoke a lot—in the year before his death often "lapsing into unconsciousness" while drinking and smoking. These episodes led to four or five incidents in which his cigarette burned holes in the rug or scarred the furniture. The defense objects that the proffered testimony is "barred by the rule against character evidence," but plaintiff's lawyer argues that Girard's account should be admitted as "evidence of habit under FRE 406." How should the court rule, and why?

Admit --It says he regularly did this and would be unconscious --This may have just been considered something that happens often --Concern if it happens regularly enough

Problem 3-H: Anna Sofer's will Anna Sofer died from injuries she suffered when run over by a bus, and her surviving husband Ira has brought a wrongful death action against the Municipal Transit Authority, operator of the bus. Under applicable law, the wrongful death claim belongs to Ira as Anna's next of kin, and he is entitled to recover for loss of companionship and expected income. Anna was a dentist with significant income. In an attempt to show that in fact Ira would have had no reasonable expectation of future financial benefit if Anna had survived, and to suggest that the quality of companionship between the two was not what might be expected between husband and wife, the Transit Authority offers in evidence a passage from Anna's will. Anna had executed the will, with the assistance of legal counsel, only a few weeks before her death. In it she had written as follows: Whereas I have been a faithful and loving wife to Ira, while he has reciprocated my tender feelings with utter cruelty, disrespect, and indifference, and whereas I have foolishly spent my best years trying hard to make him happy and to provide a comfortable home for us while he took me for granted and wasted the resources gathered by what was supposed to be our joint endeavor upon selfish and trivial pursuits every chance he got, now therefore I limit my bequest to Ira to $1, which is more than he deserves. Ira objects that Anna's will is hearsay, when offered for the stated purpose. Is it? Why or why not? To prove Ira did not suffer much from damage from Anna's death, the transit authority offers her will where she describes Ira negatively and limits his bequest to $1

Admit it --The only point that matters is what she thinks --It does not matter what her husband thinks First argue nonhearsay --If the court finds it to be hearsay offer it as state of mind exception under 803(3)

Problem 3-I: the papier mache man Zinder is prosecuted for alleged sexual assault upon eight-year-old Sharon, occurring eight months before. There is clinical evidence of assault, and Sharon testifies that Zinder assaulted her. Indeed, the defense admits that Sharon was the victim of an attack of this nature, but denies that Zinder was the assailant. To demonstrate that Zinder was the assailant, the prosecutor offers two proofs. One takes the form of an account by Officer Stalwart of the description which Sharon gave of the room to which she said she was taken by the man who assaulted her: She said the room contained what she called an "old fashioned iron bed" with "a curlicue design in the metal." She said that there were windows to the outside on two walls and a door to a closet which also had a window to the outside. She said that near the bed there was, to use her own words, "a green rocking chair and a little table with a lamp on it," and in the corner there was what she called "a papier-mâché man sitting on a wooden chair, and the man was painted red and green and blue and had a book on his lap, with his legs crossed, wearing a hat," and she said the man looked "really gay and funny, and kind of short, like a leprechaun." The other proof takes the form of a testimonial account by Officer Yeoman, who made the arrest, describing the room in which Zinder resides. Yeoman's account is independent of that given by Stalwart. That is, Yeoman was not in the courtroom while Stalwart testified; he had not talked to Sharon about the room; he described what he saw with his own eyes. These proofs seem persuasive evidence that Zinder was the culprit because the descriptions given by Sharon and Yeoman are alike in all essential details, including especially the description of the papier-mâché man. Zinder has strenuously objected to Officer Stalwart's testimony, arguing at length that "Sharon's description is hearsay, for it is offered as proof of the room in which the assault occurred, and necessarily that requires that the jury believe in the truth of her words and believe that she really thinks that the room looks as she described it." But the court rules, "Her description is not hearsay. It is offered to prove that she knows what the room looks like. I'm going to let it in." Has the court erred? Why or why not? What additional proof might the prosecutor offer in aid of the claim that Sharon's description of the room should be viewed as nonhearsay? Should out of court statements of young girl describing room where she was abused be admitted over a hearsay objection?

Admit it because it is demonstrated knowledge Someone merely saying that they can speak Chinese is hearsay But saying that someone actually spoke Chinese in class is not hearsay because it is a demonstration of an experience

Problem 5-G: "Was it a crime or just a misunderstanding? Daniel Turner is a real estate developer who needed a permit from the city of Houston to complete a multi-million-dollar development project. He is charged with attempting to bribe Mary Glenn, a member of the 9-person city council responsible for approving such permits. According to the indictment, Daniel had a meeting with Mary in which he promised to fly her to Paris for a celebration with "parting gifts" once the permit was approved. At trial, Daniel testifies that the prosecution is "just a misunderstanding," and Mary Glenn "apparently interpreted his casual remarks about celebrating the successful completion of the project as a bribe to secure her support." In its case-in-rebuttal, the prosecutor offers the testimony of Bill McCoy, also on the city council, that Daniel "offered me $50,000 prior to the vote if I would agree to support his project." The defense objects: "The prosecutor is trying to prove prior misdeeds not charged in the indictment to make his favorite propensity argument again. He knows he can't prove my client's guilt of the charged offense by any real evidence, so he wants to prove an entirely different crime for good effect. McCoy's testimony is inadmissible under FRE 404(b)." How should the court rule, and why?

Admit or not admit under FRE 404(b) --Admit because it can show a pattern of bribing to get a particular vote

Problem 12-C: The tipsy client Murphy arrives 35 minutes late for his afternoon appointment with Finch, his attorney, about a probate matter. Finch sees that Murphy has been drinking heavily, for he staggers as he is ushered into Finch's office and his breath smells of alcohol. In a slurred voice, he apologizes for being late, and explains that he had a "few drinks" with some friends and "lost track" of time. During the interview, Murphy is rambling and incoherent, and Finch finally suggests that the appointment be rescheduled. As Murphy leaves the office, Finch offers to call a taxicab, but Murphy insists on driving. Finch watches with trepidation as Murphy drives away. Two blocks down the street Murphy collides with a parked car. He is not injured and flees the scene on foot. In his later trial for hit-and-run and drunk driving, Murphy is represented by another lawyer. The state calls Finch as a witness. To what may Finch be required to testify, if Murphy claims the privilege?

Can Finch be called to testify to what he observed about Murphy's behavior, i.e. staggering, drove away --Yes, this is not a communication, this is just an observation Can Finch be called to testify about what Murphy said to him, i.e. he had a few drinks --Yes, The test is whether it was a communication for the purposes of professional legal representation Informal talks about drinking etc. is not considered something you need to know

Problem 4-E: his master's car Napton works for Ace Building Supplies, where his duties include making deliveries in a pickup truck. While working one day, Napton negligently runs over O'Brien. Napton has long been on thin ice with Ace, and a month later he loses his job for reasons unrelated to the accident. Six months later, Napton tells O'Brien that "the brakes on that truck just failed," and "I was speeding" at the time of the accident. O'Brien sues both Napton and Ace for personal injuries. At trial, O'Brien offers Napton's statement as proof that the brakes were bad and that Napton was speeding, invoking the admissions doctrine. Ace objects that it is hearsay. (Assume that, as to Ace, Napton's statement is inadmissible. Napton was not employed by Ace when he spoke, so his statement does not fit FRE 801(d)(2)(D).) How should the court rule, and why?

Can P offer the statement "I was speeding" against Napton (the driver) with limiting instruction that it is not to be used against Ace? --This can be admissible against the driver but not against the employer Can P offer the statement that "the brakes failed" against Ace? --Not admit · Not relevant, the employee may not have had any obligation and it is employer in terms of the employer

Problem 4-D: an encounter gone bad While on a skiing vacation in Vail, Colorado, a wealthy man named Kenneth Brixton struck up a conversation with a waitress named Sally Flynn in a bar. The two became friendly. They wound up walking the grounds close to the hotel, and eventually they went to Brixton's room. What happened in the room is in dispute, but two days later Flynn went to local police and said that Brixton had sexually assaulted her. On advice by a victim's advocate, Flynn went to a local hospital where a "rape kit" was used in collecting fluid samples and conducting an examination. After criminal charges of felony sexual assault were filed against Brixton, Flynn went to a lawyer and filed a civil suit for damages. In the criminal case, the lawyer for Brixton considers the possibility of entering a plea of guilty to a lesser charge of sexual contact (a misdemeanor) that would carry a lesser sentence. At a plea hearing, Brixton's lawyer knows the judge will explain the charges and range of possible sentences, and will advise Brixton that he has a right to a jury trial where the prosecutor would have to call witnesses and prove the charges against him, and that Brixton need not testify. The judge will tell Brixton that a plea of guilty waives his right to trial and leads to conviction. The judge will ask Brixton whether he committed the offense to which he is considering a plea of guilty. There Brixton will be expected to recount his version of what happened between him and Flynn in the hotel room. Would Brixton's statements in his plea hearing be admissible against him in Flynn's civil suit? What strategies should Brixton's lawyer consider? If Brixton pleads guilty and offers his version of events showing that unlawful sexual contact occurred, would that justify summary judgment for Flynn in the civil suit? Could Brixton's conviction, based on his guilty plea, become the basis for collateral estoppel in the civil suit?

Can a guilty plea in a criminal case be introduced in a later civil trial? Yes, they had their day in court What alternative should defendant use in this situation other than a guilty plea? --Nolo contender, just saying you accept the charges, not that you are guilty

Problem 4-N: "You cant offer a police report" Four days after FirstSave in Oklahoma City is robbed at 8 p.m. on Friday, June 14th, police arrest Chuck Dial and Cheryl Doran in Norman (some 20 miles from the scene of the crime). They are charged in federal court with bank robbery. Eyewitnesses place them at the scene, and their car matches the description of the vehicle apparently used by the culprits, but the proceeds of the robbery are not found, and defendants offer an alibi—they were at home in unit 5 of Stone Creek Apartments at 2640 Jefferson Street in Norman when the crime occurred, watching an episode of Big Bang Theory. On the evening of the crime, a neighbor of Chuck and Cheryl's in Norman called police to complain about excessive noise, coming from one of the apartment units in the 2600 block of Jefferson Street. A squad car came, and officers knocked on the doors of six residences to make inquiries, including unit 5 of Stone Creek Apartments. (A) Assume the police report states "06/14, 8:11PM, Knocked on apt. door at 2640 Jefferson #5; occupants Chuck Dial and Cheryl Doran answered door; said they hadn't heard unusual or loud noises." In support of their alibi defense, defendants offer this report, invoking FRE 803(8). "You can't offer a police report," objects the prosecutor, invoking the relevant language. Should the court admit the report? (B) Assume the police report states "06/14, 8:11PM, Knocked on apt. door at 2640 Jefferson #5; checked back door; looked through blinds; nobody home." To challenge the alibi defense, the prosecutor offers this this report, invoking FRE 803(8). "You can't offer a police report," objects the defense, invoking the relevant language. Should the court admit the report? Assume the police report supports defendants' alibi defense that they were home watching TV at the time of the bank robbery

Can defendants offer the police report? --Admit it because the defendant is the party who wants to use it § Can prosecutor offer the police report saying they found nobody home --Prosecutor cannot offer this --It is not a ministerial report because it is too much investigation

Problem 14-I: Cash Payment Teresa Feiler, who rents apartments to students in College Station, seeks to evict Ashley Gibson for nonpayment of rent for the month of October. Gibson testifies that she paid the October rent in cash. Counsel for Feiler gets Gibson to admit that Feiler always gave her a written receipt for her rent payments. Then Feiler's lawyer objects to Gibson's testimony, arguing that "the receipt constitutes the Best Evidence of payment, Your Honor, and she cannot testify that she made the October payment unless she has a good reason for not producing the receipt." What result, and why?

Can tenant testify that she paid the October rent in cash, when landlord claims he would have issued a receipt of such payment had it been made? · Admissible · The tenant is allowed to testify to her own conduct · What if tenant offers to testify that she made the October rent payment because she found a cancelled check for the October rent?

Problem 14-N: no pets Owner Ann Brindon seeks to evict tenant Clay Dobbs from the apartment he rents from her because he has broken his lease by keeping a dog. At the hearing, Brindon seeks to testify that the lease contains a "no pets" provision, but Dobbs' Best Evidence objection is sustained. Counsel for Brindon does not have the lease at hand, but she discovers in the file a signed letter from Dobbs acknowledging that "the lease says I cannot have any pets" and requesting special permission to keep the dog. She offers the letter, but Dobbs renews his Best Evidence objection. Brindon invokes FRE 1007. How should the court rule, and why? If Dobbs had written no letter but had orally sought exemption from the "no pets" provision from Brindon, could she prove the clause by testifying to what he said?

Can the landlord introduce tenant's letter to establish that the written lease contained a "no pets" policy? Admissible via 1006 · Does not apply to phone or oral conversations P sues security company providing security at mall. P was assaulted at mall and security guard failed to intervene to protect P. Guard testifies that "our contract with the mall prohibits us from using any physical force in connection with out duties" · Would need to produce the contract · Cannot testify to the content without producing it

Problem 14-C: there never was such an original Dan sues Eva in contract and offers in evidence what he claims to be a "photocopy of the original agreement." On inspecting the proffered photocopy and consulting with his client, counsel for Eva raises a Best Evidence objection. In proceedings in aid of the objection, Eva takes the stand, and testifies thus: Q (defense counsel): Have you examined this so-called photocopy? A (Eva): Yes, I have. Q: Is that a copy of a written agreement between you and Dan? A: I have no idea what that's a copy of. I never saw such a document in my life. We did have an agreement, and we did type it out, in a way that looks sort of like this, but this says that I promised not to go to work for any of Dan's competitors, and what we signed never contained anything like that. Our agreement, the original that we signed, never said that. Counsel for Eva then states, "Your Honor, there never was such an original as this. Therefore this purported copy—whatever it is—cannot be admitted. We've raised a genuine question about authenticity, and you cannot properly admit this purported copy—this forgery, if you want to call it what it really is." You are counsel for Dan. How can you reply to Eva's objection? If Dan can testify that "I made this photocopy from the very agreement Eva and I signed, and it accurately reproduces our contract," should the judge resolve the issue produced by the conflict between Dan and Eva, or should the jury decide it? If the jury should decide it, does that mean that the copy should be admitted? Hasn't Eva raised a "genuine issue" about the authenticity of the original?

Can the purported photocopy be introduced? 1003 allows duplicates so long as the original's authenticity is not being questioned Who decides whether the photocopy accurately represents the original? (1008) Jury determines issues relating to the existence of an asserted writing or whether other evidence of content is accurate

Problem 11-J: drunk as a skunk Prizi, a pedestrian, is seriously injured after being struck by a pickup truck driven by Davenport. The accident occurred one block from the Red Dog Saloon, where Davenport had spent the afternoon drinking. Davenport is described by one patron as "drunk as a skunk" at the time of his departure. Prizi sues the Red Dog Saloon for negligence in serving liquor to an intoxicated person whom the saloon should have foreseen was likely to drive off in a car. Prizi's action rests on a recent decision of the state supreme court recognizing a tort cause of action against tavern owners on behalf of third parties injured by patrons driving away in cars while intoxicated ("dramshop liability"). In deciding to create the new cause of action, the supreme court took "judicial notice that traveling by motor vehicle to and from a tavern is commonplace." Relying on this language, Prizi asks the trial judge to remove the issue of foreseeability from the jury. Prizi requests judicial notice that the employees of the Red Dog Saloon should have foreseen that Davenport would drive a motor vehicle upon leaving the tavern. What ruling? Ladies and gentlemen of the jury, based on the appellate case law in this state, I am taking judicial notice that the employees of the Red Dog Saloon should have foreseen that Davenport would drive a motor vehicle upon leaving the tavern

Cannot mix and match what type of notice is taken Because it was being used as an adjudicative fact in the trial case and they tried to have the judge take notice on an issue that was taken notice of as a legislative fact at the Supreme Court level

Problem 7-A: Daily Transcripts On the first day of the trial of Excel's claim against Mentor for patent infringement, counsel for plaintiff obtains a court order excluding all witnesses. Novick is the key expert witness for the defense, and he complies with the order. Just before Mentor calls Novick to testify, however, plaintiff learns that defense counsel has purchased daily transcripts of the trial, through private (and entirely proper) arrangement with the court reporter. Counsel for Excel suspects that defense counsel has at least gone over the pertinent portions of this transcript in preparing Novick for his time on the stand, and that in all likelihood Novick has actually read critical passages containing testimony given by witnesses for Excel. Plaintiff's counsel seeks to bar Novick from testifying, arguing that any sharing of transcripts violates the order of exclusion. How should the court rule, and why? Should the motion to bar Novick from testifying be allowed?

Daily transcripts cannot be prepared for a witness

Problem 10-A: the unhappy harpsichordist Atlas Moving Company advertises as follows: "We are the ones who care. Trust our people to move your most cherished possessions as carefully as you would." Glen, a professional pianist and harpsichordist, is famed for his modern interpretation of Bach's Goldberg Variations. On moving from New Haven to Los Angeles to become pianist in residence at the University of Southern California, Glen hired Atlas to move his valued antique harpsichord, along with other household furniture. The month was April, and one Larson (an experienced mover for Atlas) picked up and loaded Glen's possessions on the 12th and drove straight to Los Angeles, arriving there on the 16th. When the shipment arrived, Glen discovered a deep crack in the inner casing of the harpsichord, which severely impaired its tonal quality. Glen sues Atlas for $20,000, supporting his damage claim with expert testimony. Glen himself testifies that he plays the instrument nearly every day and that it was in perfect condition when Atlas picked it up in New Haven. Atlas introduces the deposition of Keenan, another accomplished keyboard artist in New Haven. The transcript contains Keenan's testimony that he played Glen's harpsichord in New Haven and noticed the crack in the casing at the time, along with impaired tonality. At the close of the evidence, Glen requests the court to instruct the jury that if it finds that the harpsichord was undamaged when Larson picked it up, it must find Atlas responsible for damage to the instrument. Should the instruction be given? "Ladies and gentlemen of the jury, I instruct you that if you find the harpsichord was undamaged when Atlas picked it up and was returned in a damaged condition, you must find Atlas was negligent in causing the damage."

Give the presumption What matters is if it was damaged or undamaged when it was given initially This is a jury question because they were challenging a basic fact

Problem 12-G: the reluctant lawyer A federal grand jury is deciding whether to indict Kastin for fraud based on his alleged use of the mail to send false and misleading brochures soliciting investments in a real estate partnership. Walters, a real estate attorney who had represented Kastin, is called before the grand jury to describe discussions with Kastin relating to the brochure prior to mailing. The government is trying to show that Kastin knew there were false statements in the brochure, but Walters claims the attorney-client privilege on behalf of Kastin and refuses to testify. The court orders Walters to answer, finding that the discussions fit the future crime or fraud exception to the privilege. Assume that Walters has a reasonable basis for believing that the court has erred. Does he have an ethical obligation to protect Kastin by defying the order, at the cost of being held in contempt, so the merits of the privilege claim can be reviewed? What if Walters is unwilling to be held in contempt? Should Kastin be allowed to intervene and to appeal the order requiring Walters to disclose?

If a lawyer believes the court erred in overruling the privilege claim of the client, does the attorney have an ethical obligation to defy the disclosure order and refuse to reveal the confidential communication, even if it means the attorney will be held in contempt? --DR 4-101 Preservation of Confidences and Secrets of a Client ---(b) except when permitted under DR 4-101(c), a lawyer shall not knowingly reveal a confidence or secret of his client. ---(c) a lawyer may reveal: ----(2) confidences or secrets when permitted under disciplinary rules or required by law or court order

Problem 5-S: "We can talk, but you have to sign a waiver first" DEA Agents suspect that Greg Marky and Herb Stovall are involved in the drug trade in the suburbs of Dayton, Ohio (the acronym stands for Drug Enforcement Administration). After staking out Stovall's house, agents conclude that Stovall is running a meth lab. They often see Marky arrive and depart, and agents think Marky is a dealer and that he purchases meth from Stovall for resale on the street. The agents accost Stovall but don't shut down his lab, in hopes of catching "bigger fish." Asked about Marky's visits, Stovall tells agents that Marky is "one of the biggest dealers in Dayton." The agents ask Stovall to set up a "controlled buy" in which an agent pretending to be a customer will purchase meth from Marky for cash. This transaction goes as planned, and DEA agents arrest Marky. Represented by appointed counsel, Marky sits down with an Assistant U.S. Attorney (AUSA) and some of the agents to talk. The AUSA tells Marky he's facing "upwards of 20 years in federal prison" for this transaction and others that they have found out about. "We can talk," says the AUSA, "but we make no promises of a deal, and you have to sign this first." He hands Marky a waiver form that says "if these negotiations do not end in a plea, or lead to a plea that the court does not accept, or that is abrogated by either side for any reason, anything Marky says in these negotiations can be used in a trial against him." Advised by counsel to sign, Marky does so and admits knowing that "the package contained meth." Encouraged to "tell us all you know about the drug trade in Dayton," Marky identifies more than ten other people, including Stovall, and describes many drug transactions involving those people. But negotiations break down without a deal when the AUSA decides Marky can't be trusted. He has told them that his visit to Stovall's place was "the first time I'd been there in over a month," which conflicts with what the agents had seen in their stakeout. Marky is charged with selling meth in the controlled buy set up with Stovall's help. Marky takes the stand in his own defense, and testifies that he "didn't know the package contained meth," and that he thought Stovall was making "plastic explosives" and that the agent who bought the package was involved in a mining venture and needed the explosives for that purpose. Over defense objection, the prosecutor calls a DEA agent to testify that Marky admitted in their conversation with the AUSA that he knew the package he got from Stovall contained meth. The defense objects that "this testimony is barred by FRE 408," but the prosecutor replies that "Marky waived his right to exclude what he said in their conversation." How should the court rule, and why?

Is this really plea bargaining? The court is split

Problem 3-E: whose corn? John Lord leased part of his farm to Frank Cartwright for payment in kind, in the amount of 40 percent of the corn Cartwright could grow. Cartwright borrowed money from First State Bank, giving the bank a security interest in the part of his crop not promised to Lord, but ultimately defaulted on the loan. The bank repossessed a double crib of corn from the farm and sold it to Dan Prager. Lord sues Prager and the bank for conversion, claiming that the corn that the bank repossessed and sold was Lord's share of Cartwright's crop. As proof that the corn belonged to him, Lord testifies that he and Cartwright went to the field one day, and Cartwright "pointed out the corn in the double crib and said, 'Mr. Lord, this double crib of corn is your share for this year, and it belongs to you, sir.'" The bank and Prager object that Lord's description of what Cartwright said and did was hearsay. As proof that the corn in the double crib was indeed the corn covered by its security interest, the bank offers testimony by its loan officer that "when we came out to see about selling the corn, Cartwright told us the corn in the double crib was his." Lord objects that the loan officer's description of what Cartwright said and did was hearsay. What result on these hearsay objections, and why? Assume that if the bank got the wrong corn, Prager and the bank are liable to Lord for conversion.

Lord offers to testify that Cartwright told him "this double crib of corn is your share for this year and it belongs to you, sir" --This is admissible because it involves the important words necessary under the law of personal property Loan officer of bank offers to testify that Cartwright said the corn in the double crib is mine --Do not admit because this does not speak to the actual transfer of property, just a claim that it is his

Problem 12-F: the disclosure was inadvertent Milton & Associates, a general contractor, sued Beacon Building Supply (BBS) in federal court alleging breach of contract in connection with the construction of a large office complex. BBS was represented in the litigation by Carol Brock, a partner in the Frank/Carlton law firm, and associate Kevin Marlow. Milton filed a discovery request for all the BBS internal emails relating to the construction project. To assist in identifying such emails, and screening out any that might be privileged, Brock hired a technology consulting company, Discovery Manager Inc. Using its own specially designed software, Discovery Manager was able to identify 63,200 internal BBS emails relating to the project. Then in consultation with Brock and Marlow, Discovery Manager sought to screen out and remove any of those emails that might be protected from disclosure by the attorney-client privilege or work product doctrine. It did so by isolating all emails addressed to or received from attorneys at Frank/Carlton. The software identified 840 such emails, and they were removed. Marlow then did a manual review of the remaining 62,360 emails, largely by sampling, and identified 38 additional emails that he removed as privileged or work product. Once the protected emails were removed and entered on a privilege log, the remaining 62,322 emails were turned over to Milton on May 14.On June 20, counsel for Milton notified Brock that 62 emails were found in the disclosed emails that appeared to be subject to the attorney-client privilege. Brock responded the same day stating that the disclosure was inadvertent and she would be back in touch shortly. Over the next three weeks, Brock and Marlow were involved in eight depositions in the case, as well as a summary judgment motion brought by another defendant. On July 18, Brock called Milton's counsel and learned that the 62 privileged emails involved correspondence with Frank/Carlton attorneys that were attachments to other emails so the connection to Frank/Carlton did not appear in the headers of the emails. On July 22, Brock sent a letter to Milton's counsel asserting the privilege claim and demanding return of the emails. Milton refused, contending that any claim of privilege had been waived by the failure of BBS to conduct a proper privilege review prior to disclosing the emails and by its delay in demanding their return. On August 1, Brock filed a motion seeking a court ruling that the 62 emails are privileged and compelling their return. How should the court rule?

Should BBS lose its claim for privilege protection and return of documents because it never conducted a post-disclosure review --No, ACN said you don't have to do that Assume FRE 502 controls and no nonwaiver agreement had been signed. Would BBS lose its claim for privilege protection because it conducted an inadequate privilege review and therefore did not take reasonable steps to prevent the disclosure? --The court in this case did not find the waiver --The company did not set up the computer program to look for attached documents so it was critical Assume no nonwaiver agreement had been signed and FRE 502 controls. Would BBS lose its claim for privilege protection and return of documents because it delayed more than one month in asserting the privilege claim and demanding return of the emails? --Some courts would say this is too long and they should have acted earlier --Some would say it is okay given the pressures of trial

Problem 12-H: the babysitter's testimony Charley and his wife Edith were returning home late one night from a party where they both had too much to drink. Charley, who was driving, failed to see Max, an elderly pedestrian who was crossing the street at a properly marked crosswalk. He struck and killed him. Charley panicked and, over Edith's screams, fled the scene without stopping and returned to their home. After they entered their house, Pam, the babysitter, asked Edith why she was so upset. She tearfully replied, "Because Charley ran over someone on the way home." After a police investigation, criminal charges are brought against Charley for vehicular manslaughter. At trial, Edith refuses to testify for the prosecution, claiming the testimonial privilege. The prosecutor then calls Pam as a witness at trial to relate what Edith had said to her. The court finds that Edith's statement to Pam qualifies under the excited utterance exception to the hearsay rule, but both Charley and Edith object to Pam's testimony, again asserting the testimonial privilege. What ruling?

Should Edith's excited utterance be admitted against Charley at his trial for vehicular manslaughter? --the spousal privilege only applies to the two spouses, not to anyone that might have been told their information

Problem 2-B: was he going too fast? On an open stretch of two lane highway in Nevada, Jay Gadsby, traveling eastbound in a red Z Car with racing stripe, collided with Roy Reinhart, headed westbound in a pickup truck with gunrack. Both Jay and Roy were killed instantly. The road was straight, with noonday sun bright overhead, and afternoon thermal winds had not yet picked up-in short, driving conditions were optimal. Physical facts yield no clues as to the cause of the accident. In her wrongful death action against Gadsby's estate, Roy's widow offers testimony by another eastbound driver, Hill who was the first one to get to the accident, that 30 miles west of the point of collision the red car had overtaken him going at least 80 miles per hour. The defense objects, arguing the testimony is irrelevant when offered as proof that Jay was speeding at the time of the accident, at least in absence of further proof that Gadsby likely continued to travel at the rate observed for the 30 miles between the sighting and the point of impact. Is this evidence relevant on the question of Gadsby's speed at the time of impact? Should the judge admit the evidence only if the proponent offers additional proof to satisfy the condition suggested by defendant?

Should Hill be allowed to testify to Gadsby's speed at a distance of .3 miles prior to the point of collision? Yes this should be admitted Should he be allowed to testify if he was 30 miles away? No he should not be allowed Should he be allowed to testify if he was 300 miles away? No he should not be allowed

Problem 12-E: hit and run to your lawyer While driving his car home late at night, Dale runs over and kills a pedestrian named Baltes. Dale panics and speeds away from the scene. Media reports indicate that police are looking for the driver. They have a description of Dale's car but not the license number or enough detail to identify him. Dale feels remorse and retains Arnold as his attorney. He asks Arnold to negotiate a plea bargain but not to reveal his identity if negotiations fail. The prosecutor refuses to negotiate without knowing who Arnold represents. The Baltes family learns from the district attorney that Arnold represents the hit-and-run driver. The family files a wrongful death suit against "John Doe" and seeks a court order compelling Arnold to reveal his client's identity. Arnold claims the identity is privileged. How should the court rule?

Should the court require the identity to be disclosed? --Do not have to disclose it, it would be disclosing what the client talked about Disclosing identity would reveal the communication with the client

Problem 9-A: it was my impression In the trial of Cox for unlawful detonation of explosives, arising from the firebombing of cars, the prosecutor calls defendant's former girlfriend Carter. She testifies that Cox told her twice that he knew someone who would blow up cars for $50 and that he showed her a newspaper account of one of the bombings giving rise to the charges. During the state's case-in-chief, the following exchange occurs: Q [prosecutor]: Did Mr. Cox admit being involved in this bombing? A [Carter]: He never actually said that, you know, he had blown it up, but it was my understanding by his mentioning that he had a friend and showing me the article that it was my impression when we were done talking that he was involved in having blown it up. Ms. Dreeves: The defense objects to that, Your Honor. It calls for opinion and speculation, and her so-called impression isn't helpful in understanding her testimony or determining the facts in issue in this case. Put together the prosecutor's argument that Ms. Carter's testimony satisfies FRE 701. How will the defense reply to this argument? How should the court rule, and why? It was my impression when we were done talking that he was involved in having blown up the car

The court did not admit this --When a statement looks too speculative the court will not admit

Problem 5-O: was he served? Manuel Gutierrez is charged with illegal entry into the country after previous deportation. At trial, the government must prove that he was served a warrant of deportation and a letter in his native language warning him of the penalties of reentry. As part of its proof, the government calls Agent Lesher from the United States Immigration and Customs Enforcement Division of the Department of Homeland Security (ICE). Lesher himself did not serve Gutierrez, nor did he ever serve deportation warrants. But he has worked for ICE in a variety of positions for 12 years, serving on traffic check, line watch, and as warrant officer. If permitted, he would describe procedures followed by officers in carrying out deportations and the preparation and service of the forms. He would testify that normally an immigration officer picks up the "deportee," fills in blanks on the back of the warrant, signs it as witness, obtains the right thumb print of the deportee on the back, and hands him the letter and a copy of the warrant. In this case, no signature appears on the warrant, although it is otherwise filled out and has defendant's thumb print. During defense voir dire, Agent Lesher testifies that he has never executed such a warrant and his knowledge of the procedures comes from "what I have heard from detention officers" and from knowing "normal rules and processes of deportation." Defendant then objects that Agent Lesher's testimony cannot establish service in this case. Should Lesher be permitted to testify?

The court held that even if you learned about the practice from third parties, he is a weak foundation witness, but still found to be adequate to testify In this case, it was admitted Don't have to have an expert to demonstrate that there was a routine practice

Problem 4-J: "I felt this sudden pain" Fifty-five-year-old Eldon Sanders was employed as a pumper and well treater by Texas Oil (he was a 23-year veteran with the company). His job required him manually to load 30-gallon containers of chemicals onto a pickup truck and then drive around to wells in the field, unload the containers, and pour the contents down the wells. From time to time he also had to move pumps weighing 50 to 100 pounds, lifting them onto the truck and unloading them. On the morning in question, Sanders had his usual coffee and doughnut at a cafe and drove the pickup into the fields at about 7:00 a.m. Two hours later he was seen en route to the "Chase Lease" to treat wells. At 10:00 a.m. he returned to town and went home, where he complained somewhat breathlessly to his wife Eleanor, "I felt this sudden pain just a few minutes ago when I had to lift one of those 30-gallon cans out on the Chase." Eleanor drove her husband to the office of their family physician Dr. Hillier, who took an electrocardiogram. There was no indication of heart damage, though Sanders' blood pressure was elevated. Hillier administered a tranquilizer and put Sanders to bed for observation. At first he rested easy and seemed better, but in the afternoon he became uncomfortable, and at 5:00 p.m. he died of acute myocardial infarction (heart attack). Eleanor Sanders sues Liberty Insurance Company, the workers' compensation carrier for Texas Oil, seeking benefits as surviving widow. She testifies that she was surprised to see her husband come home at that hour, and that he "never comes home at that time." She offers to testify to what he told her, and counsel invokes the excited utterance exception. Liberty objects, arguing that "there's no proof of an exciting event here, other than the statement itself, and letting the statement prove the condition on which admissibility depends would be bootstrapping." How should the court rule, and why? Is Sanders statement to his wife admissible to prove he felt a sudden pain and that it happened at work? Is this bootstrapping?

This is bootstrapping, it proves the fact needed to be proven Can get around it by proving some other evidence that she went to work that day to prove that he was at work

Problem 5-R: I used his stuff Martin Rackly is charged with passing counterfeit bills. His lawyer Kent Slavin sets up an appointment with Assistant U.S. Attorney Amy Norton to discuss the situation. Slavin meets Norton and two Secret Service agents in the Federal Building, where Norton tells Slavin, "I'm not prepared to make a plea bargain because I haven't had a chance to study the case." Slavin replies that "Rackly's involvement in the counterfeiting operation is marginal, but he can help you get the guys you really want. So if you think you're interested in a deal, I think Rackly will cooperate with you." Thereafter Slavin calls Norton's office and sets up another meeting in the Federal Building. He and Rackly attend, as do the Secret Service agents, but nobody from Norton's office is there. The agents deliver Miranda warnings, but Slavin protests that "We're here to talk about a plea, not a confession." On Slavin's advice, Rackly refuses to sign a Waiver of Rights Form (which would acknowledge receipt of Miranda warnings and say that "no promises have been made" and that Rackly "waives his right against self-incrimination"). He then makes a detailed statement, which includes the following crucial points: "Brody was my source. He had the bills, and I passed quite a few of them. I used his stuff at Wolf Brothers and Champion Electronics." Ensuing telephone conversations between Slavin and Norton result in a draft plea agreement, but in the end Rackly does not sign. He is tried for passing bills at Wolf Brothers and Champion Electronics. The prosecutor offers in evidence Rackly's statements admitting those transactions, but the defense objects: "Those are plea bargaining statements, and they should be excluded under Rule 410." What result, and why? Are Rocky's statements to the Secret Service admissible at trial or are they excluded by 410?

This was a mistake by both attorneys 410 does not include the police but only includes statements made to an attorney

Problem 5-F: "He came running in all hunched over" Danzey and Gore are charged with bank robbery. The prosecutor calls eyewitnesses who describe the crime. One who was outside saw two men in ski masks and gloves emerge from a white car and enter the bank. Another in the bank said one of the men came running in all hunched over and vaulted the counter, taking the money rays from the teller and stuffing them into a bag. A third witness in an apartment two blocks away saw the perpetrators arrive at about 9 AM in separate cars, one white and the other brown. They returned a short time later, got into the brown car, and left the scene. An eyewitness can identify Danzey as one of the robbers, but nobody can identify the other. As proof that the other was Gore, the government introduces his confession of involvement in eight similar robberies, all committed between 9AM and 11 AM, all using stolen cars of contrasting light and dark colors. In his confession, Gore said he always wears a ski mask and gloves and always runs hunched over and jumps over the counter.

To prove Gore was the other robber, can the government offer evidence that he committed prior bank robberies under similar circumstances? --If they find the circumstances sufficiently unique and similar they can let it in --Proving the defendant's modus operandi But within 404(b) --Permitted uses: this evidence may be admissible for another purpose, such as proving motive, opportunity, intent, preparation, plan, knowledge, identity, absence of mistake, or lack of accident --Proves identify ---Identify the identity of this bank robber based on previous circumstances ---Has to be very unique and sufficient circumstances What if the prior circumstances were that in a prior robbery D had a ski mask and wore gloves? --Is this sufficient to establish identity or a "signature" crime?

Problem 12-D: a failed venture Samuel and Thomas consult with lawyer Ullman on forming a partnership to import computer chips from Asia. The three discuss the venture, to be called Accu-Chip, Limited. Samuel indicates that he has "lots of contacts in Tokyo and Taipei who will supply quality chips," but he acknowledges that "many cheap chips available in those markets don't meet American specifications." Thomas provides most of the start-up money and some expertise on customs law and import fees. The two sign a partnership and go into business. But Accu-Chip falls on hard times. It has difficulty with the chip quality; the market for chips declines because of technological changes and falling prices. 1. In a suit against Accu-Chip and the partners (Samuel and Thomas), dissatisfied customer Vanden alleges breach of implied warranty and misrepresentation of the quality of Accu-Chip products. Vanden takes the deposition of Ullman (who represents the defendants) and asks, "What did your clients tell you about the quality of the chips they intended to import?" On behalf of defendants, Ullman invokes the attorney-client privilege and refuses to answer. Vanden moves to compel answers. What result and why? 2. Samuel retains Wilson as counsel and files suit against Thomas for alleged breach of the partnership contract and an accounting for partnership profits. At trial, Samuel takes the witness stand and proposes to testify to what Thomas said in Ullman's office when the partnership was formed. For Thomas, Ullman invokes the attorney-client privilege. What result, and why? 3. In a criminal prosecution against Samuel and Thomas for alleged conspiracy to evade customs inspections and import fees, Wilson represents Samuel and Ullman represents Thomas. The four meet together to coordinate strategy. During the meeting, Samuel angrily reminds Thomas that "you were supposed to be the expert on customs and taxes." Thomas replies, "I carried out that end and handled it as well as anybody could." At trial, Samuel proposes to testify to what Thomas said, but Ullman objects, raising the attorney-client privilege. What result, and why?

What about joint clients? --Communications among them protected against outsiders but not in litigation among themselves They lose the privilege if they have a dispute with each other

Problem 4-K: "He says he'll kill me" The prosecutor has evidence that defendant Otto Neff was shaking down Paul Quade, who was found dead of knife wounds in a park. Neff is a small-time tough guy, and the prosecutor thinks he was collecting "protection" money from Quade, who balked and was killed for resisting. At trial the prosecutor calls Quade's friend Roy Sarnak, who will testify (if permitted) that during the period in question Quade once told him: Neff is after me again. He says he'll kill me and my family if I don't pay protection. I've already paid him $5,000, and I'm trying to steer clear of him, and I need help but I just don't know what to do. Neff raises a hearsay objection. The prosecutor invokes FRE 803(3). If the charge against Neff is extortion, what arguments and counterarguments do you expect on the admissibility of what Quade said? What if the charge against Neff is murder? If the charge were murder, would it make a difference that Neff claims self-defense?

What is Quade (the victim) saying? --"I am afraid of Neff" In extortion prosecution, government offers victim's statement: "I am afraid of Neff" --Admit it because it is a then-existing mental, emotional, or physical condition --Prove defendant put the victim in fear In a murder prosecution prosecutor offers evidence that before his death Quade told a friend "I'm afraid Neff is going to kill me" --Do not admit because state of mind is not relevant in a murder trial

Problem 6-E: the $800,000 jury error When Baker loses his farm to the bank in a foreclosure, he turns against his lawyer, Henderson, and brings a malpractice suit alleging that Henderson failed properly to advise him on the matter of redeeming the farm. At the conclusion of the case, the court delivers the following instruction on the proper measure of damages:" The Plaintiff's damages are the fair market value of the farm minus the redemption costs plus the mortgage balance on the property." The evidence indicates that fair market value was about $500,000, that costs of redemption would have been about $10,000, and that the outstanding balance on the mortgage was about $400,000. During deliberations, the jury foreman sends a note to the judge asking whether the jury must follow this formula in calculating damages. With approval by both lawyers, the judge answers in the affirmative. After returning a verdict for Baker in the amount of $890,000, the jury is dismissed. The court enters judgment on the verdict. Immediately Henderson's lawyer gets permission from the court to interview jurors. Within days, he obtains affidavits from all 12, saying that they understood that they were to add the mortgage balance to fair market value (rather than subtracting the former from the latter). Henderson seeks a new trial, arguing that the jury made an $800,000 mistake. If the judge agrees that the jury erred, should she grant a new trial? Does FRE 606(b) allow such affidavits? Could Henderson have asked the trial judge, at the time the jury announced its verdict, to ask the jury how it understood the instruction? § Should an affidavit by jurors explaining how they made the erroneous calculation be allowed?

Would not allow the affidavit to be shown where the jury calculated the costs It was inadmissible cannot bring in evidence to show how the jury reached their verdict

Problem 12-B: the bail jumper Woodburn is out on bail pending trial on an indictment charging him with bank robbery. One of the conditions is that he stay in touch with his attorney Nash and appear at the time set for trial. Woodburn fails to appear, and the government seeks an indictment against Woodburn for bail jumping. The government subpoenas Nash before the grand jury and asks him whether he advised his client of the time and place of trial. Nash declines to answer, asserting the privilege on behalf of Woodburn. The government obtains a court order requiring Nash to answer, but he continues in his refusal and is held in contempt. What ruling in an appeal by Nash from his contempt citation? Can attorney be forced to answer grand jury's question about whether he told the client about the trial date?

Yes This is not professional legal advice, the attorney is just acting as a messenger

Problem 5-M: Death on the Highway Lance Teel and his wife were driving westbound on Highway 46 when they collided with a car driven by Paul Finney, in which his wife was a passenger. All four were killed, and no eyewitness saw the accident. The cars collided at the intersection of the highway and a county road, and the patrolman who examined the scene concluded that the Finney vehicle was entering the highway from the north and was in the process of turning left (eastward) when it was struck. The accident occurred at 8:00 a.m. on a cool cloudy gray September morning, when visibility was only about one-quarter of a mile. The Teel estate sues the Finney estate for wrongful death. The patrolman testifies about the scene, and the court allows him to give his opinion as an expert in accident reconstruction on the probable speed and positions of the cars at the time of impact. Because the accident looked like one that could happen only if one of the two drivers was at fault, and because there was no direct proof on this point, the Teel estate seeks to prove that Lance Teel was exercising due care. It calls witnesses Budge and Frese, who are prepared to testify that Lance Teel was "a good, careful driver." The Finney estate objects that the proffered evidence "is just proof of character, which cannot be admitted to show conduct on a particular occasion." How should the court rule, and why?

do not admit, this is general and character evidence


Kaugnay na mga set ng pag-aaral

algebra 2a - unit 1: factoring and solving quadratics lesson 1-4

View Set

HESI A2 - Critical Thinking- Rationale

View Set

Exam 5 Practice Questions (Varcarolis Chapters 18, 20, 26)

View Set

Chapter 9 Reading Quiz and Homework

View Set

CVHS - Economics - Chapter 4 Study Guide

View Set

Chapter 2: Managing People and Work

View Set